0% found this document useful (0 votes)
33 views62 pages

Norcet 9 Mains Toppers Mcqs 100 Mcqs All Subjects #2 by Nprep Nursing

Uploaded by

shaikhalaf178
Copyright
© © All Rights Reserved
We take content rights seriously. If you suspect this is your content, claim it here.
Available Formats
Download as PDF, TXT or read online on Scribd
0% found this document useful (0 votes)
33 views62 pages

Norcet 9 Mains Toppers Mcqs 100 Mcqs All Subjects #2 by Nprep Nursing

Uploaded by

shaikhalaf178
Copyright
© © All Rights Reserved
We take content rights seriously. If you suspect this is your content, claim it here.
Available Formats
Download as PDF, TXT or read online on Scribd
You are on page 1/ 62

OBSTETRICS 1.

Normal – no fetal distress


Q. The following image shows a 2. Borderline – repeat sampling within 30
bluish-purple discolored uterus with blood minutes
infiltrating into the myometrium. This 3. Abnormal – immediate delivery required
condition is most consistent with: 4. Invalid – repeat test immediately
Correct Answer: 3. Abnormal – immediate
delivery required

Rationale:
Normal fetal scalp blood pH: > 7.25 → No
fetal distress.
Borderline pH: 7.20–7.25 → Repeat test
within 30 minutes.
Abnormal (fetal distress): < 7.20 →
Indicates fetal acidosis → Immediate
delivery.

Q. The image below shows a uterus with


vertical compression sutures applied over
1. Rupture uterus
the uterine body to control postpartum
2. Couvelaire uterus
hemorrhage (PPH). This surgical procedure
3. Uterine atony
is known as:
4. Placenta accreta

Correct Answer: 2. Couvelaire uterus

Rationale:
Couvelaire uterus (uteroplacental apoplexy)
occurs due to severe abruptio placentae.
Blood from the retroplacental clot infiltrates
into the myometrium and beneath the
uterine serosa, giving the uterus a 1. B-Lynch suture
bluish/purplish ecchymotic appearance. 2. Hayman suture
It does not rupture but may lose 3. Cho’s multiple square sutures
contractility. 4. Hysterectomy
Differentiates from uterine rupture (full
thickness tear with bleeding into peritoneal Correct Answer: 2. Hayman suture
cavity).
Rationale:
Q. Fetal scalp blood sampling was
performed during labor. The pH result is
7.18. How should the nurse interpret this
finding?
such as in front of a mirror, while bathing,
and lying down

Rationale:
Nursing officer’s role → To educate on
when and how to do BSE.
When: Once a month, ideally 5–7 days after
menstruation (breasts less tender).
How:
In front of mirror → observe for asymmetry,
dimpling, nipple changes.
While bathing → soapy fingers glide easily,
making lumps easier to feel.
Hayman’s suture is a uterine compression Lying down → breast tissue spreads evenly,
suture technique used for controlling atonic aiding detection of deep lumps.
postpartum hemorrhage when medical Daily or irregular practice is unnecessary
management fails. and may reduce compliance.
Similar to B-Lynch but simpler, as it does
not require opening the uterus. Q. The nursing officer is assessing a
Vertical sutures are passed through the woman’s breast during a clinical
uterine wall (anterior to posterior) and tied examination (see image). This clinical sign
above the fundus, compressing the uterus. is most commonly associated with which
Preserves fertility and avoids hysterectomy. condition?

Q. A nursing officer is conducting a health


education session on breast
self-examination (BSE). Which of the
following instructions should she include to
ensure proper technique and timing?

1. Perform BSE only if pain or lump is felt in


the breast
2. Perform BSE monthly, 5–7 days after
1. Fibroadenoma
menstruation, using methods such as in
2. Carcinoma of breast
front of a mirror, while bathing, and lying
3. Mastitis
down
4. Fibrocystic disease
3. Perform BSE daily in the morning before
bathing
Correct Answer: 2. Carcinoma of breast
4. Perform BSE during menstruation when
breasts are swollen
Rationale:
Peau d’orange (orange peel appearance)
Correct Answer: 2. Perform BSE monthly,
occurs due to lymphatic obstruction from
5–7 days after menstruation, using methods
infiltrating carcinoma of the breast.
The obstruction causes localized edema,
making skin tethered around hair follicles
→ dimpling.
Mastitis shows redness and pain but not
peau d’orange.
Fibroadenoma / Fibrocystic disease cause
lumps or cystic changes, not skin dimpling.

Q. You are shown the following obstetric


image during a difficult delivery:

Wood’s Corkscrew (B) = rotation of


posterior shoulder.
Rubin’s (D) = pressure on posterior aspect
of fetal shoulder to rotate.

Q. Which hormone is primarily responsible


for initiating the ovarian cycle by
stimulating the growth of ovarian follicles?

1. Progesterone
2. Follicle Stimulating Hormone (FSH)
3. Luteinizing Hormone (LH)
4. Estrogen
This procedure is known as:
Correct Answer: 2. Follicle Stimulating
1. McRoberts Maneuver Hormone (FSH)
2. Wood’s Corkscrew Maneuver
3. Zavanelli Maneuver Rationale:
4. Rubin’s Maneuver The ovarian cycle begins with the follicular
phase, initiated by FSH, which stimulates
Correct Answer: 3. Zavanelli Maneuver the growth and maturation of ovarian
follicles.
Rationale: LH surge later triggers ovulation.
Zavanelli Maneuver: Involves flexing and Estrogen is produced by developing
pushing the delivered fetal head back into follicles, providing feedback.
the vagina to perform an emergency Progesterone dominates in the luteal phase
cesarean section. to prepare the endometrium.
Used as a last-resort measure in severe
shoulder dystocia. Q. A nurse is giving health education to a
McRoberts (A) = maternal hips hyperflexed pregnant woman diagnosed with placenta
to abdomen.
previa. Which of the following instructions but often migrates upward as the uterus
is most appropriate? enlarges (“placental migration”).
Therefore, diagnosis before 20 weeks is not
1. Perform regular vaginal examinations to reliable.
monitor bleeding A definitive diagnosis is made by
2. Immediately report any episode of ultrasound at 28–32 weeks, when the
vaginal bleeding likelihood of spontaneous migration is
3. Engage in regular sexual intercourse to minimal.
promote placental circulation At labor, diagnosis may be incidental if not
4. Increase physical activity to prevent detected earlier, but standard protocol
complications confirms at 28–32 weeks.

Correct Answer: 2. Immediately report any Q. A nurse is caring for 4 obstetric clients in
episode of vaginal bleeding the antenatal ward. Which client should be
given highest priority?
Rationale:
In placenta previa, the placenta covers the 1. A 32-week pregnant woman with painless
cervical os → risk of painless, recurrent vaginal bleeding and a soft uterus
vaginal bleeding. 2. A 36-week pregnant woman with severe
Vaginal examinations are contraindicated headache, blurred vision, and +3
(they can trigger massive hemorrhage). proteinuria
Sexual intercourse and strenuous activity 3. A 30-week pregnant woman with mild
are restricted to prevent bleeding. ankle edema
Health education focuses on promptly 4. A 12-week pregnant woman with morning
reporting any bleeding, avoiding vaginal sickness
exams, abstaining from intercourse, and
limiting strenuous activity. Correct Answer: 2. A 36-week pregnant
woman with severe headache, blurred
Q. At which gestational age is it most vision, and +3 proteinuria
appropriate to make a definitive diagnosis
of a low-lying placenta (placenta previa)? Rationale:
Option a: Painless vaginal bleeding with
1. 12–14 weeks soft uterus = placenta previa (serious, but if
2. 18–20 weeks stable → monitor & prepare).
3. 28–32 weeks Option b: Severe headache, blurred vision,
4. At the time of labor +3 proteinuria = severe preeclampsia /
impending eclampsia → life-threatening,
Correct Answer: 3. 28–32 weeks highest priority.
Option c: Mild edema = common in
Rationale: pregnancy, non-emergency.
In early pregnancy (first & early second Option d: Morning sickness at 12 weeks =
trimester), a low-lying placenta is common expected.
Among abnormal situations (placenta 2. Cystocele
previa vs severe preeclampsia), severe 3. Rectocele
preeclampsia is the greater immediate 4. Vault prolapse
threat to maternal life → priority.
Correct Answer: 2. Cystocele
Q. Use of combined oral contraceptive pills
(OCPs) provides protection against which Rationale:
cancers and increases the risk of which Cystocele (prolapse of the anterior vaginal
cancer? wall with bladder descent) is the most
common type of genital prolapse.
1. Protection – Ovarian & Endometrial Uterine prolapse is common but less
cancer; Risk – Breast & Cervical cancer frequent than cystocele.
2. Protection – Cervical & Breast cancer; Rectocele involves posterior vaginal wall
Risk – Ovarian & Endometrial cancer prolapse and is less common.
3. Protection – Colorectal & Breast cancer; Vault prolapse occurs after hysterectomy
Risk – Ovarian & Cervical cancer and is less frequent overall.
4. Protection – Ovarian & Cervical cancer;
Risk – Endometrial & Breast cancer Q. A 28-year-old woman presents with a
recurrent, painful swelling at the vaginal
Correct Answer: 1. Protection – Ovarian & introitus diagnosed as a Bartholin’s cyst.
Endometrial cancer; Risk – Breast & What is the definitive management?
Cervical cancer
1. Incision and drainage
Rationale: 2. Marsupialization
Protective effect: OCPs reduce lifetime 3. Word catheter placement
ovulation and unopposed estrogen 4. Oral antibiotics alone
exposure → lower risk of ovarian and
endometrial cancer. Correct Answer: 2. Marsupialization
Increased risk: Long-term OCP use is
associated with a slight ↑ risk of breast and Rationale:
cervical cancers. Incision & drainage → only temporary, high
Also protective against colorectal cancer recurrence.
(secondary benefit). Word catheter → useful for first episode or
non-recurrent cases.
Q. A 55-year-old postmenopausal woman Marsupialization → creates a permanent
presents with a sensation of something opening, definitive treatment for recurrent
coming down from the vagina. On Bartholin cyst/abscess.
examination, the most common type of Antibiotics → not definitive, only adjunct if
genital prolapse is identified. Which of the infection is present.
following is the most common type of
genital prolapse? Q. According to WHO reference values for
semen analysis, which of the following
1. Uterine prolapse
parameters correctly represent the lower 1. Anterovaginal retractor
reference limits for normal semen? 2. Leech Wilkinson cannula
3. Rubin’s cannula
1. Volume ≥ 1.4 mL, Progressive motility ≥ 4. Hegar’s dilator
42%, Morphology ≥ 4%
2. Volume ≥ 2.0 mL, Progressive motility ≥ Correct Answer: 2. Leech Wilkinson
30%, Morphology ≥ 10% cannula
3. Volume ≥ 1.0 mL, Progressive motility ≥
50%, Morphology ≥ 14% Rationale:
4. Volume ≥ 1.5 mL, Progressive motility ≥ 1. Leech Wilkinson cannula is used during
40%, Morphology ≥ 6% Hysterosalpingography (HSG) or
chromopertubation to introduce
Correct Answer: 1. Volume ≥ 1.4 mL, dye/contrast into the uterine cavity.
Progressive motility ≥ 42%, Morphology ≥ 2. It has a conical tip with screw-like
4% grooves to fix it firmly into the cervix and
prevent leakage of dye.
Rationale: 3. Mainly employed in tubal patency testing.
1. Volume: ≥ 1.4 mL 4. Different from Rubin’s cannula, which is
2. Total sperm count: ≥ 39 million/ejaculate a simple metal cannula without screw
3. Sperm concentration: ≥ 16 million/mL mechanism.
4. Progressive motility: ≥ 42%
5. Vitality: ≥ 54% live spermatozoa Q. A 42+3 weeks pregnant woman is
6. Normal morphology: ≥ 4% admitted for induction of labor. On
7. pH: ≥ 7.2 ultrasound, amniotic fluid index (AFI) is 4
8. Leukocytes: > 1 million/mL may indicate cm, and CTG shows variable decelerations.
infection Which of the following is the most
appropriate next step in management?
Q. Identify the instrument shown in the
image, which is commonly used in 1. Continue expectant management with
evaluation of female infertility. daily NST
2. Induce labor with prostaglandins
immediately
3. Perform cesarean section
4. Give IV fluids and repeat CTG after 1 hour

Correct Answer: 3. Perform cesarean


section

Rationale:
1. Post-term pregnancy (≥ 42 weeks) carries
risk of placental insufficiency &
oligohydramnios.
2. AFI ≤ 5 cm = oligohydramnios, increases Q. Which of the following statements about
risk of cord compression & fetal distress. Cervavac (Human Papillomavirus vaccine)
3. CTG showing variable decelerations = is correct?
evidence of cord compression &
compromised fetus. 1. It protects against all types of HPV
4. In such case → immediate delivery is infections
required, and C-section is safer than 2. It is indicated for prevention of cervical
induction. cancer and genital warts
5. Expectant management or induction is 3. It is given as a single-dose vaccine only
contraindicated when there is fetal distress. 4. It is used to treat existing cervical cancer

Q. Which of the following statements about Correct Answer: 2. It is indicated for


the Pap smear (Papanicolaou test) is prevention of cervical cancer and genital
correct? warts

1. It is used to screen for endometrial Rationale:


cancer in postmenopausal women 1. Cervavac is an HPV vaccine developed in
2. It detects cervical cancer and India, targeting the high-risk HPV types 16
precancerous lesions and 18, responsible for the majority of
3. It is performed by collecting a blood cervical cancers.
sample 2. It helps prevent cervical precancerous
4. It is a diagnostic test for ovarian cancer lesions, cervical cancer, and genital warts.
3. It is a prophylactic vaccine, not a
Correct Answer: 2. It detects cervical treatment for existing infections or cancer.
cancer and precancerous lesions 4. Recommended schedule:
Two doses for girls aged 9–14 years
Rationale: Three doses for individuals aged 15–26
1. Pap smear is a screening test for cervical years or immunocompromised patients
cancer, aiming to detect dysplastic or 5. Early vaccination before sexual debut
precancerous changes in cervical epithelial provides maximum protection.
cells.
2. Cells are collected from the Q. Implantation occurs in a definite
transformation zone of the cervix using a sequence during the receptive phase of the
spatula, brush, or cytobrush. endometrium. Arrange the steps in correct
3. It does not detect endometrial or ovarian order and identify the timing (window of
cancer. implantation):
4. Early detection via Pap smear allows
timely intervention and reduces cervical (A) Apposition of blastocyst with
cancer mortality. endometrium
5. Screening recommendation: (B) Adhesion of blastocyst to endometrium
Women aged 21–29: every 3 years (C) Invasion (penetration) of trophoblast
Women 30–65: every 3 years (Pap alone) or into endometrium
every 5 years (co-testing with HPV)
Options: Correct Answer: 3. 8 weeks
1. (A) → (B) → (C); Day 20–24 of menstrual
cycle (Window of implantation) Rationale:
2. (B) → (A) → (C); Day 10–14 of cycle In the first 4–6 weeks, the embryo looks
3. (C) → (B) → (A); Day 14–18 of cycle more like a curved structure with
4. (A) → (C) → (B); Day 24–28 of cycle pharyngeal arches and tail bud, not
distinctly human.
Correct Answer: 1. (A) → (B) → (C); Day By the end of 8 weeks, organogenesis is
20–24 of menstrual cycle nearly complete, the head is large, limb
buds are developed, eyes and ears are
Rationale: distinct → the embryo starts to look
Apposition (Step 1): The blastocyst loosely recognizably human.
aligns with the endometrial epithelium. This After 8 weeks, it is called a fetus (from 9th
is the first, unstable contact. week onwards).
Adhesion (Step 2): The blastocyst firmly
attaches to the endometrial epithelium Q. Which drug is most effective for
through adhesion molecules (e.g., integrins, prevention of recurrent preterm labour in
selectins). high-risk pregnancy?
Invasion (Step 3): The trophoblast cells
penetrate the endometrial epithelium and 1. Oxytocin
invade the stroma to establish 2. Progesterone
maternal–fetal circulation. 3. Ergometrine
Timing (Attaration / Window of 4. Indomethacin
Implantation):
Occurs Day 20–24 of a 28-day menstrual Correct Answer: 2. Progesterone
cycle (about 6–7 days after ovulation /
fertilization). Rationale:
During this phase, the endometrium is most Progesterone maintains uterine quiescence
receptive due to progesterone dominance, by:
pinopodes development, and increased Inhibiting prostaglandin synthesis
adhesion molecule expression. Reducing myometrial contractility
Outside this window, implantation usually Promoting cervical stability
fails. Given as 17-α hydroxyprogesterone
caproate (IM, weekly) or micronized vaginal
Q. At which week of intrauterine life does progesterone.
the embryo begin to resemble a human Indicated in:
form? History of previous preterm birth
Short cervix detected on ultrasound
1. 4 weeks Other drugs (like nifedipine, atosiban,
2. 6 weeks indomethacin, β-agonists) are used as
3. 8 weeks tocolytics to acutely suppress contractions,
4. 12 weeks not for long-term prevention.
Q. Which of the following drugs is 4. Placental abruption
recommended for the prevention of
pre-eclampsia (PIH) in high-risk pregnant Correct Answer: 1. Premature closure of the
women? ductus arteriosus
1. Aspirin (low-dose)
2. Nifedipine Rationale:
3. Methyldopa Indomethacin is a NSAID used as a
4. Magnesium sulfate tocolytic to inhibit preterm labor (by
reducing prostaglandin synthesis).
Correct Answer: 1. Aspirin (low-dose) Prostaglandins keep the ductus arteriosus
patent in fetal circulation.
Rationale: Third-trimester use can lead to:
Low-dose Aspirin (75–150 mg daily, started Premature closure of ductus arteriosus →
after 12–16 weeks and continued till 36 pulmonary hypertension in fetus
weeks): Oligohydramnios (due to decreased fetal
Inhibits platelet thromboxane synthesis renal perfusion)
Improves uteroplacental circulation First and second trimester use generally
Proven to reduce risk of pre-eclampsia in avoids this risk, but prolonged use is still
high-risk women (e.g., chronic cautioned.
hypertension, diabetes, renal disease, prior Other adverse effects: fetal renal
PIH). impairment, necrotizing enterocolitis in
Calcium supplementation (1–1.5 g/day) is preterm infants if used late.
also recommended in women with low
dietary calcium intake. Q. Look at the attached diagram of the
Methyldopa & Nifedipine: Used for placenta:
treatment of hypertension in pregnancy, not
for prevention.
Magnesium sulfate: Used for seizure
prophylaxis in eclampsia, not prevention of
PIH itself.

So:
Drug to prevent PIH: Low-dose Aspirin
Supplement: Calcium (in deficient women)

Q. Use of indomethacin during pregnancy,


especially in the third trimester, can lead to
Which of the following conditions is
which of the following complications?
illustrated in the image?
1. Premature closure of the ductus
1. Velamentous cord insertion
arteriosus
2. Battledore placenta
2. Neural tube defects
3. Placenta previa
3. Maternal hypoglycemia
4. Circumvallate placenta
Correct Answer: 2. Battledore placenta Uterine artery: Main supply, arises from
internal iliac artery.
Rationale: Ascending branch: Uterine body & fundus,
Battledore placenta (Marginal insertion): anastomoses with ovarian artery (important
Umbilical cord inserts at the margin of the collateral).
placenta, resembling a paddle Descending branch: Cervix and upper
(“battledore”). vagina.
Usually asymptomatic, but may Ovarian artery: From abdominal aorta,
occasionally cause mild fetal growth supplies ovaries and some uterine fundus,
restriction. but not main supply.
Velamentous insertion: Cord inserts into Vaginal artery: Supplies vagina and lower
fetal membranes, vessels run unprotected cervix; minor contribution to uterus.
→ higher risk of vasa previa and fetal Superior gluteal artery: Gluteal region; no
hemorrhage. role in uterine supply.
Placenta previa: Placenta covers internal
cervical os. Q. During normal pregnancy, which
Circumvallate placenta: Placental edges are statement about uteroplacental blood flow
folded back; associated with preterm labor, is correct?
bleeding.
1. Blood flow is constant throughout
Q. Which of the following correctly pregnancy at ~50 mL/min
describes the main uterine blood supply 2. Uteroplacental blood flow increases
and its important anastomosis? progressively and can reach 500–700
mL/min at term
1. Uterine artery (from internal iliac) → 3. It is primarily supplied by the ovarian
ascending branch to fundus, descending artery only
branch to cervix; anastomoses with ovarian 4. It is independent of maternal blood
artery pressure
2. Ovarian artery (from internal iliac) →
supplies fundus and body; anastomoses Correct Answer: 2. Uteroplacental blood
with uterine artery flow increases progressively and can reach
3. Uterine artery (from external iliac) → 500–700 mL/min at term
main supply to uterus; anastomoses with
ovarian artery Rationale:
4. Superior gluteal artery → main supply to Uteroplacental circulation:
uterus; no anastomoses Provides oxygen and nutrients to the fetus.
Progressive increase: From ~50 mL/min in
Correct Answer: 1. Uterine artery (from early pregnancy → 500–700 mL/min at term.
internal iliac) → ascending branch to Increase occurs due to:
fundus, descending branch to cervix; Vasodilation of uterine spiral arteries
anastomoses with ovarian artery Decrease in vascular resistance
Arterial supply: Mainly from uterine artery,
Rationale: with contribution from ovarian artery.
Dependent on maternal blood pressure and (E) Uterine massage
cardiac output: Hypotension can reduce
flow. Options:
1. (E) → (A) → (C) → (B) → (D)
Q. At what gestational age can fetal heart 2. (B) → (E) → (A) → (C) → (D)
sounds (FHS) be first heard using a Doppler 3. (B) → (A) → (E) → (C) → (D)
device in a normal pregnancy? 4. (A) → (E) → (B) → (C) → (D)

1. 6 weeks Correct Answer: 2. (B) → (E) → (A) → (C) →


2. 8 weeks (D)
3. 10–12 weeks
4. 16–18 weeks Rationale:
Step 1: Assess cause (4 Ts) – Always first
Correct Answer: 3. 10–12 weeks to direct management.
Step 2: Uterine massage – Immediate
Rationale: bedside measure to promote uterine
Fetal heart activity: contraction.
Detected by ultrasound: As early as 5–6 Step 3: Uterotonic drugs – Oxytocin or
weeks (via transvaginal US, cardiac flicker). misoprostol to treat atony.
Heard by Doppler: Usually 10–12 weeks, Step 4: Tranexamic acid – Administered
becomes more reliable after 12 weeks. within 3 hours to reduce bleeding.
Heard by Pinard stethoscope (fetal Step 5: Surgical/interventional procedures –
stethoscope): Around 16–20 weeks Only if bleeding persists despite
depending on maternal habitus and fetal conservative measures.
position. Key point for exams: The sequence tests
Clinical significance: Early detection your understanding of prioritization, timing,
confirms viable intrauterine pregnancy. and escalation in PPH management.

Q. A 28-year-old woman develops primary Q. Which of the following statements about


postpartum hemorrhage after vaginal lochia is correct?
delivery. Arrange the following steps in the
most appropriate sequence for
1. Lochia is the normal postpartum vaginal
management: discharge, consisting only of blood.
2. Lochia changes in character and color
(A) Administer uterotonic drugs (oxytocin, over time, reflecting uterine involution.
misoprostol) 3. Lochia is only seen after cesarean
(B) Assess and manage underlying cause (4 section, not vaginal delivery.
Ts: Tone, Trauma, Tissue, Thrombin) 4. Persistent lochia beyond 1 week is
(C) Give tranexamic acid within 3 hours of always pathological.
birth
(D) Perform surgical interventions (uterine Correct Answer: 2. Lochia changes in
balloon, compression sutures, artery character and color over time, reflecting
ligation, hysterectomy) uterine involution
Rationale:
Lochia is the postpartum uterine discharge
composed of blood, decidual tissue,
mucus, and epithelial cells.
Timeline & types:
1. Lochia rubra (0–3 days): Bright red,
mainly blood.
2. Lochia serosa (4–10 days): Pink or
brown, contains serous fluid and
leukocytes.
3. Lochia alba (11–28 days, can last up to 6 4.
weeks): Yellowish-white, mostly leukocytes Correct Answer: 1. Kielland forceps
and mucus.
Rationale:
Physiological significance: Reflects uterine
involution and tissue healing. Kielland forceps:
Persistent foul-smelling or excessively Sliding shanks allow rotation of the fetal
heavy lochia may indicate infection or head in malposition (e.g., occiput
retained products, but normal lochia can transverse).
last several weeks. Wrigley forceps: Short, low-cavity, mainly
for low-vertex or breech deliveries.
Q. Which of the following forceps is Piper forceps: Designed for breech
specifically designed for rotational deliveries to assist after-coming head.
maneuvers of the fetal head? Simpson forceps: General high-cavity
forceps, not specifically for rotation.

1.

2.

3.
FUNDAMENTAL OF NURSING CPAP may feel uncomfortable, as the
Q. A patient with COPD and CO₂ retention is patient must exhale against continuous
admitted to the ICU. The doctor advises pressure.
non-invasive ventilation. Which mode is BiPAP is more comfortable since the
most appropriate and why? exhalation pressure (EPAP) is lower.
5. In this case (COPD with CO₂ retention):
1. CPAP – because it provides one The patient needs assistance in ventilation
continuous pressure throughout inspiration to remove CO₂.
and expiration Therefore, BiPAP is the correct choice.
2. BiPAP – because it provides different
pressures during inspiration and expiration Key Exam Line:
3. CPAP – because it improves both CPAP = Oxygenation support
oxygenation and ventilation BiPAP = Oxygenation + Ventilation support
4. BiPAP – because it gives continuous (Best for COPD with hypercapnia)
pressure and is cheaper than CPAP

Correct Answer: 2. BiPAP – because it


provides different pressures during
inspiration and expiration, helping in CO₂
elimination

Rationale (Point-wise):
1. Pressure Levels
CPAP delivers a single continuous pressure
during both inspiration and expiration.
BiPAP delivers two different pressures:
IPAP (Inspiratory Positive Airway Pressure)
→ Assists with inhalation.
EPAP (Expiratory Positive Airway Pressure)
→ Keeps alveoli open during exhalation. Q. A nurse is caring for a patient
2. Oxygenation vs Ventilation undergoing intracavitary radiation therapy.
CPAP → Improves oxygenation only by Suddenly, the radiation source becomes
preventing alveolar collapse. dislodged. Which is the most appropriate
BiPAP → Improves both oxygenation and immediate nursing action?
ventilation, because IPAP supports
inspiration and helps in blowing off CO₂. 1. Pick up the source with gloved hands
3. Indications and place it in the linen bag
CPAP → Mainly for Obstructive Sleep 2. Use long-handled forceps to place the
Apnea, pulmonary edema, neonatal RDS. source in a lead container kept in the room
BiPAP → Used in COPD with hypercapnia 3. Call the physician first before handling
(CO₂ retention), respiratory failure, and the source
patients intolerant to CPAP. 4. Allow the client’s family to assist in
4. Comfort locating the source
Correct Answer: 2. Use long-handled suspected aplastic anemia. Which of the
forceps to place the source in a lead following is considered the best site for
container kept in the room bone marrow biopsy in adults?

Rationale: 1. Sternum
Never touch the source with bare hands – it 2. Posterior superior iliac spine
can cause radiation injury. 3. Tibial tuberosity
Lead container and forceps must always be 4. Anterior iliac crest
available in the client’s room for emergency
management. Correct Answer: 2. Posterior superior iliac
The physician should be informed after spine
securing the source.
Visitors should not be allowed until the Rationale:
source is accounted for. Posterior superior iliac spine (PSIS) is the
The nurse should also reassure the client preferred site for bone marrow biopsy in
after ensuring safety. adults because:
It provides a broad, safe area.
Q. During colonoscopy, which position is Risk of injury to vital organs is minimal.
most commonly used for the patient to Adequate marrow can be obtained.
facilitate easy passage of the colonoscope? Sternum → used for aspiration only, not
biopsy (thin bone, risk of mediastinal
1. Supine position injury).
2. Left lateral (Sim’s) position Tibial tuberosity → used in infants/young
3. Prone position children, not adults.
4. Lithotomy position Anterior iliac crest → can be used as an
alternative site, but PSIS remains the safest
Correct Answer: 2. Left lateral (Sim’s) and best.
position
Q. While working in a hospital ward, a nurse
Rationale: notices that a fire has broken out in the
The left lateral (Sim’s) position is most utility area. What should be the first action
commonly used during colonoscopy. according to the fire safety protocol?
In this position, the patient lies on the left
side with hips and knees flexed. 1. Extinguish the fire immediately
It straightens the sigmoid colon and 2. Call the fire department
rectum, making insertion of the 3. Rescue patients from immediate danger
colonoscope easier and safer. 4. Close all doors and windows
Supine position may be used if the
procedure is difficult, while lithotomy is Correct Answer: 3. Rescue patients from
mainly for anorectal surgeries. immediate danger

Q. A hematologist plans a bone marrow Rationale:


biopsy for a 35-year-old patient with
Fire safety in hospitals follows the RACE 1. Pulling the skin and subcutaneous tissue
principle: laterally before injection.
R → Rescue persons in danger 2. Inserting the needle into the muscle and
A → Alarm/activate the fire alarm injecting medication slowly.
C → Contain the fire by closing 3. Waiting 10 seconds before withdrawing
doors/windows the needle.
E → Extinguish fire if safe to do so 4. Releasing the displaced skin so it returns
Thus, the first priority is patient safety, i.e., to normal position, sealing the medication
rescue → then alarm → contain → in the muscle.
extinguish. This technique prevents irritation, leakage,
and tissue staining.
Q. A nurse is preparing to administer an
intramuscular injection of iron dextran, Q. Identify the instrument.
which is known to be irritating to
subcutaneous tissue. To prevent
medication leakage into subcutaneous
tissue and reduce skin staining, which
technique should the nurse use?
1. Sphygmomanometer
1. Standard intramuscular injection 2. JVP manometer
2. Z-track technique 3. Pulmonary artery catheter
3. Subcutaneous injection 4. Stethoscope
4. Intradermal injection
Correct Answer: 2. JVP manometer
Correct Answer: 2. Z-track technique
Rationale:
Rationale: JVP manometer is used to measure jugular
The Z-track technique is used for venous pressure, reflecting right atrial
intramuscular injections of medications that pressure.
are irritating or can stain subcutaneous The patient is usually positioned at 30–45°,
tissue (e.g., iron dextran, some with the manometer level aligned at the
antipsychotics). sternal angle.
Steps include: Elevated JVP can indicate right heart
failure, fluid overload, or constrictive
pericarditis.

Other instruments:
Sphygmomanometer → measures blood
pressure.
Pulmonary artery catheter → measures
pulmonary pressures invasively.
Stethoscope → auscultation of heart and
lungs, not pressure.
Q. A patient develops cardiogenic shock Rationale:
following a massive myocardial infarction. 1. The bladder scanner is a non-invasive
Which of the following changes is expected portable ultrasound device.
in central venous pressure (CVP)? 2. It is used to measure bladder volume and
post-void residual urine (PVR).
1. CVP decreases 3. Helps in early detection of urinary
2. CVP increases retention.
3. CVP remains normal 4. Avoids unnecessary catheterization,
4. CVP fluctuates randomly reducing the risk of UTI and patient
discomfort.
Correct Answer: 2. CVP increases 5. Commonly indicated in neurogenic
bladder, post-operative cases, and urinary
Rationale: retention assessment.
Cardiogenic shock → due to severe left or
right ventricular failure → decreased Q. A central venous catheter (CVP line) has
cardiac output. one lumen that is not in use. According to
Right ventricular failure → blood backs up standard guidelines, how often should this
into venous system → increased CVP. unused lumen be flushed to maintain
patency?
Other notes:
Hypovolemic shock → CVP decreases 1. Every 6 hours
Septic shock → early CVP may be 2. Every 12 hours
low/normal, later may rise with fluid 3. Every 24 hours
resuscitation 4. Only when accessed
Obstructive shock (e.g., tamponade,
tension pneumothorax) → CVP increases Correct Answer: 3. Every 24 hours

Q. Identify the type of device shown in the Rationale:


image. 1. Prevents Occlusion – Flushing once
every 24 hours prevents clot formation or
blockage within the lumen.
2. Maintains Patency – Keeps the lumen
patent and ready for immediate use in case
of emergency.
3. Reduces Infection Risk – Less frequent
manipulation (compared to flushing every
1. Invasive urinary catheterization device few hours) lowers the chance of introducing
2. Portable bladder scanner infection.
3. Cystoscope 4. Evidence-Based Protocols – Most
4. Uroflowmetry device institutional guidelines recommend flushing
unused lumens at least once every 24
Correct Answer: 2. Portable bladder hours.
scanner (ultrasound-based)
5. Balance of Safety & Efficacy – Daily 4. If misaligned – Too high → falsely low
flushing ensures line functionality without pressure; too low → falsely high pressure.
excessive handling or risk of 5. Standardization – Used universally as the
complications. reference for hemodynamic monitoring in
critically ill patients.
Q. While setting up invasive hemodynamic
monitoring (e.g., central venous pressure or Q. The central venous pressure (CVP)
arterial line), the transducer must be zeroed reading should be measured by the nurse at
at the phlebostatic axis. This anatomical which point of the respiratory cycle for
reference point corresponds to: accuracy?

1. Mid-clavicular line at the level of the 1. At the peak of inspiration


second intercostal space 2. At the end of expiration
2. Mid-axillary line at the level of the fourth 3. At the beginning of expiration
intercostal space 4. During forced inspiration (deep breath)
3. Mid-axillary line at the level of the fifth
intercostal space Correct Answer: 2. At the end of expiration
4. Mid-clavicular line at the level of the fifth
intercostal space Rationale :
1. Respiratory effect on CVP: Intrathoracic
Correct Answer: 2. Mid-axillary line at the pressure changes during breathing can
level of the fourth intercostal space alter CVP readings
2. Inspiration effect: Negative intrathoracic
Rationale pressure during inspiration causes CVP to
fall.
3. Expiration effect: At end-expiration,
intrathoracic pressure is closest to
atmospheric pressure, giving the most
accurate reflection of right atrial pressure.
4. Standard practice: Therefore, CVP is
consistently measured at end-expiration to
avoid respiratory variation errors.

Q. A nurse is caring for a client with a


1. Definition – The phlebostatic axis is the closed chest drainage system and observes
external reference point used to align the that tidaling (fluctuation) in the water-seal
transducer with the level of the right atrium. chamber has stopped. Which of the
2. Landmark – It is located at the following is the most likely cause?
intersection of the mid-axillary line and the
fourth intercostal space. 1. The chest tube is kinked or obstructed
3. Clinical Importance – Ensures accurate 2. There is a continuous air leak in the
pressure readings (CVP, PA pressures, system
arterial pressures). 3. The suction pressure is set too low
4. The suction pressure is set too high 3. Simple face mask: Provides 40–60% FiO₂.
4. Venturi mask: Delivers precise FiO₂
Correct Answer: 1. The chest tube is (24–50%), useful in COPD, but not
kinked or obstructed maximum concentration.
5. Clinical importance: Ambu bag is
Rationale: primarily used in resuscitation or
1. Tidaling = normal: Fluctuation of water in emergency ventilation where immediate
the water-seal chamber occurs with high oxygen delivery is critical.
inspiration and expiration when the system
is patent. Q. Match the correct patient position for
2. Stopped tidaling: Suggests that the tube each airway procedure:
is no longer patent — due to kinking, Procedures:
dependent loop, or clot obstruction.
3. Alternative possibility: Tidaling may also 1. ETT intubation
stop if the lung has fully re-expanded, but 2. ETT extubation
obstruction must always be ruled out first. 3. ETT suctioning
4. Air leak: Would be indicated by
continuous bubbling, not absence of Options:
tidaling. A. Semi-Fowler’s position (30–45°)
5. Suction changes: Alter bubbling in the B. Supine with neck slightly extended
suction control chamber, not tidaling in the (“sniffing position”)
water-seal chamber. C. Supine with slight head elevation
Select the correct sequence:
Q. Which oxygen delivery method can
provide the highest concentration of 1. 1-B, 2-A, 3-C
oxygen when connected to an oxygen 2. 1-C, 2-B, 3-A
source? 3. 1-A, 2-C, 3-B
4. 1-B, 2-C, 3-A
1. Venturi mask
2. Ambu bag Correct Answer: 1. 1-B, 2-A, 3-C
3. Nasal cannula
4. Simple face mask Rationale :
Intubation: Sniffing position for proper
Correct Answer: 2. Ambu bag alignment of airway axes.
(Bag-Valve-Mask) Extubation: Semi-Fowler’s to reduce
aspiration risk and aid coughing.
Rationale : Suctioning: Supine with slight head
1. Ambu bag (BVM): When connected to elevation for drainage and safety.
100% oxygen and used with a tight seal, it
can deliver close to 100% FiO₂, making it Q. Which of the following is considered the
the highest among non-invasive methods. most reliable method to confirm correct
2. Nasal cannula: Delivers 24–44% FiO₂ at placement of an endotracheal tube (ETT)?
low flow.
1. Observation of chest rise and Correct Answer: 2. Swan-Ganz catheter
auscultation of breath sounds
2. Fogging of the ETT during exhalation Rationale:
3. Capnography (end-tidal CO₂ monitoring)
4. Condensation in the ETT

Correct Answer: 3. Capnography (end-tidal


CO₂ monitoring)

Rationale :
1. Capnography: Provides continuous,
quantitative confirmation of ETT placement
in the trachea; the presence of consistent
CO₂ waveform confirms tracheal, not
esophageal, placement.
2. Chest rise & auscultation: Useful but less
reliable; breath sounds may be misleading,
especially in obese or noisy patients.
3. Fogging/condensation: Can occur in
esophagus or trachea; not definitive.
4. Clinical importance: Early and accurate
confirmation of ETT prevents hypoxia,
aspiration, and ventilatory complications.
The Swan-Ganz catheter is a pulmonary
Q. Identify the catheter shown in the X-ray artery catheter inserted via central vein →
image, commonly used to measure RA → RV → pulmonary artery.
pulmonary artery pressures and cardiac It is used to measure pulmonary artery
output: pressure, pulmonary capillary wedge
pressure, cardiac output, and mixed venous
oxygen saturation.
CVC (option A) stops at the SVC/RA
junction, Hickman (option C) is for
long-term chemotherapy, and PD catheter
(option D) is intra-abdominal.
Identify the catheter shown in the X-ray
image, commonly used to measure Q. A nursing officer is checking the
pulmonary artery pressures and cardiac endotracheal (ET) tube cuff pressure with a
output: cuff manometer. The reading shows 25 cm
H₂O.
1. Central venous catheter What should be the next appropriate action
2. Swan-Ganz catheter by the nurse?
3. Hickman catheter
4. Peritoneal dialysis catheter
1. Immediately deflate the cuff to release Rationale:
pressure Colorimetric CO₂ detector changes color in
2. Inflate the cuff further to 35–40 cm H₂O response to exhaled CO₂.
3. Maintain the cuff pressure, document the Purple → Yellow = “Yes it’s in the airway”
finding in the nursing notes, and continue (trachea).
monitoring No color change = possible esophageal
4. Remove the ET tube and reintubate intubation.
This method provides a rapid bedside
Correct Answer: 3. Maintain the cuff confirmation of ET tube placement, though
pressure, document the finding in the continuous capnography is more reliable.
nursing notes, and continue monitoring
Q. World Hand Hygiene Day is observed
Rationale: every year on:
Normal ET tube cuff pressure: 20–30 cm
H₂O (to prevent tracheal ischemia & 1. 5th May
aspiration). 2. 15th October
In this case, 25 cm H₂O is within the normal 3. 7th April
range. 4. 12th December
Priority action: maintain as is, document,
and continue periodic monitoring (every 8 Correct Answer: 1. 5th May
hours or as per protocol).
Overinflation (>30 cm H₂O) → risk of Rationale:
tracheal mucosal damage. World Hand Hygiene Day: Observed on 5th
Underinflation (<20 cm H₂O) → risk of May each year, led by WHO, with the slogan
aspiration & leak. “SAVE LIVES: Clean Your Hands”.
The date (5/5) is chosen to represent 5
Q. After intubating a 40-year-old patient in fingers on each hand.
the emergency room, the nursing officer It emphasizes hand hygiene in healthcare
attaches a colorimetric CO₂ detector to settings to prevent healthcare-associated
confirm endotracheal tube placement. The infections.
detector changes color from purple to Global Handwashing Day: Different from the
yellow. above, it is celebrated on 15th October
What does this finding indicate? every year, focusing on handwashing with
soap in the community and schools to
1. Tube is in the esophagus prevent infectious diseases like diarrhea
2. Tube is correctly placed in the trachea and pneumonia.
3. Tube cuff is overinflated
4. Tube needs to be advanced further into Q. A nursing officer is performing hand
the bronchus hygiene before patient care. What is the
recommended minimum duration for:
Correct Answer: 2. Tube is correctly placed
in the trachea A. Alcohol-based hand rub
B. Medical hand washing with soap and Cloth towels (A) → risk of
water cross-contamination.
Air dryers (B) → less effective, may
1. Hand rub 10–15 sec; Hand wash 20 sec disperse microbes into the environment.
2. Hand rub 20–30 sec; Hand wash 40–60 Air-drying by shaking hands (D) →
sec unhygienic and not recommended.
3. Hand rub 40–60 sec; Hand wash 20–30
sec Q. A 40-year-old man is admitted with
5. Hand rub 1 min; Hand wash 2 min pulmonary tuberculosis. The nursing officer
implements airborne precautions. Which of
Correct Answer: 2. Hand rub 20–30 sec; the following is the most appropriate
Hand wash 40–60 sec infection control measure for this patient?

Rationale: 1. Place patient in a negative-pressure room


Alcohol-based hand rub → WHO and use N95 respirator
recommends 20–30 seconds to cover all 2. Place patient in a single room with
hand surfaces. surgical mask for staff
Medical (hygienic) hand wash with soap & 3. Place patient in a well-ventilated room,
water → Recommended 40–60 seconds. gloves only required
Shorter durations are ineffective, and 4. No isolation needed, only standard
longer durations are unnecessary in routine precautions are enough
clinical practice.
Correct Answer: 1. Place patient in a
Q. After performing proper hand washing, negative-pressure room and use N95
what is considered the best method for respirator
drying hands in a healthcare setting?
Rationale:
1. Use of personal cloth/towel Airborne precautions are required for
2. Use of hot air dryer diseases transmitted via droplet nuclei <5
3. Use of disposable paper towels μm (e.g., TB, measles, varicella, COVID-19
4. Shaking hands in air until dry in aerosol-generating procedures).
Key measures:
Correct Answer: 3. Use of disposable paper Patient placement in negative-pressure
towels isolation room (AIIR).
N95 respirator (or equivalent) for healthcare
Rationale: workers.
Disposable paper towels are the best Limit patient transport; if necessary, patient
method in healthcare settings because wears a surgical mask.
they: Options B & C are insufficient.
Dry hands quickly Option D ignores transmission risk
Reduce microbial contamination
Minimize cross-infection risk Q. A 6-year-old child with acute
lymphoblastic leukemia is undergoing
chemotherapy and has developed severe Correct Answer: 3. 0.5 mm Pb
neutropenia. The nursing officer prepares
to implement reverse isolation. What is the Rationale:
primary purpose of reverse isolation in this
patient?

1. To protect healthcare workers from


patient’s infection
2. To prevent cross-infection between
patients in the ward
3. To protect the immunocompromised
patient from infections carried by others
4. To reduce the workload of nursing staff
by limiting patient contact Lead aprons are used to protect healthcare
workers from scatter radiation during
Correct Answer: 3. To protect the radiological procedures.
immunocompromised patient from Standard thickness for diagnostic X-rays:
infections carried by others 0.5 mm lead equivalent provides adequate
protection.
Rationale: Thinner aprons (<0.5 mm) may not
Reverse isolation (protective isolation) is adequately reduce radiation exposure;
used for patients with severe thicker aprons (>0.5 mm) offer slightly more
immunosuppression (e.g., neutropenia, protection but are heavy and reduce
post-transplant, extensive burns). mobility.
Goal = protect patient from external Aprons should cover front and sides, with
pathogens (staff, visitors, environment). thyroid collars as needed.
Measures: strict hand hygiene, PPE for
visitors, no fresh flowers/raw fruits, sterile Q. A 60-year-old male with chronic kidney
environment. disease presents with lethargy,
It is opposite of standard isolation (which hypotension, and diminished deep tendon
protects others from the infected patient). reflexes. Laboratory results show serum
magnesium 4.5 mg/dL.
Q. In a radiology department, a nursing Which of the following is the most
officer wears a lead apron while assisting appropriate initial management for
during X-ray procedures. What is the hypermagnesemia in this patient?
recommended minimum thickness of a lead
apron to provide effective protection? 1. Administer intravenous calcium
gluconate
1. 0.05 mm Pb 2. Initiate potassium supplementation
2. 0.25 mm Pb 3. Administer intravenous magnesium
3. 0.5 mm Pb sulfate
4. 1.0 mm Pb 4. Start high-dose loop diuretics only
Correct Answer: 1. Administer intravenous 30 minutes before transfusion if prescribed,
calcium gluconate or IV immediately before transfusion

Rationale: Rationale:
Hypermagnesemia occurs commonly in Patients with a history of transfusion
renal failure or excessive magnesium reactions may require pre-medication to
intake. reduce risk of allergic or febrile reactions.
Clinical features: lethargy, hypotension, Oral medications should be given 30
nausea, vomiting, bradycardia, minutes prior to allow absorption.
diminished/absent reflexes, respiratory IV medications can be administered
depression. immediately before starting the transfusion.
Initial management: Close monitoring during the transfusion is
Calcium gluconate IV → stabilizes cardiac also essential.
membrane and neuromuscular function. Overdosing (D) or ignoring history (B, C)
Remove magnesium sources. can cause harm.
Enhance elimination via diuretics (if
kidneys work) or hemodialysis in renal Q. A nursing officer is inserting a
failure. nasogastric (NG) tube in a conscious adult
Magnesium administration (C) worsens the patient. The tube has reached the first black
condition; potassium (B) is unrelated. measurement marking near the back of the
throat.
Q. A nursing officer is preparing to start a What is the recommended next step to
blood transfusion for a patient with a advance the NG tube safely?
history of mild allergic reactions during
previous transfusions. 1. Force the tube gently down without any
What is the recommended nursing action additional maneuver
regarding premedication before starting the 2. Instruct the patient to swallow or sip
transfusion? water, and if resistance is met, rotate the
tube downward and toward the closer ear
1. Administer acetaminophen or 3. Pull the tube back completely and retry
diphenhydramine orally 30 minutes before from the other nostril
transfusion if prescribed, or IV immediately 4. Flex the patient’s head backward and
before transfusion push the tube
2. Start the transfusion immediately without
premedication, regardless of history Correct Answer: 2. Instruct the patient to
3. Only monitor the patient during swallow or sip water, and if resistance is
transfusion, no premedication is required met, rotate the tube downward and toward
4. Give double the usual dose of the closer ear
antihistamine to prevent any reaction
Rationale:
Correct Answer: 1. Administer When the tube reaches the pharynx,
acetaminophen or diphenhydramine orally swallowing or sips of water helps guide it
into the esophagus rather than the trachea.
Resistance may indicate contact with the What is the recommended method to
posterior pharyngeal wall; gentle rotation secure the catheter tubing to prevent
and angling toward the closer ear helps trauma or accidental dislodgement?
passage.
In intubated or semiconscious clients, 1. Tape the tubing loosely to the bedside
flexing the head toward the chest facilitates rail
esophageal entry. 2. Leave the tubing unsecured to allow free
Forcing the tube (A, D) increases risk of movement
trauma or airway insertion. Pulling back 3. Secure the tubing to the inner thigh using
entirely (C) is unnecessary if proper an stabilization device.
technique is followed. 4. Tie the tubing in a knot near the catheter
insertion site
Q. A nursing officer is caring for a patient
with a tracheostomy tube in the ICU. Correct Answer: 3. Secure the tubing to the
Which of the following is the most inner thigh using an stabilization device
important bedside preparedness for this
patient? Rationale:

1. Keep suction apparatus only


2. Keep a resuscitation (Ambu) bag at the
bedside at all times
3. Keep extra gloves and mask only
4. Keep an extra nasogastric tube

Correct Answer: 2. Keep a resuscitation


(Ambu) bag at the bedside at all times
Securing the catheter tubing prevents:
Rationale: Accidental pulling or dislodgement
Patients with endotracheal or tracheostomy Trauma to the urethra
tubes are at risk of airway obstruction, Infection risk
accidental decannulation, or respiratory Inner thigh is preferred to reduce tension
distress. on the urethra.
A resuscitation (Ambu) bag must always be StatLock or similar agency-approved
readily available to provide immediate securement devices are recommended over
manual ventilation if the patient cannot tape or knots.
breathe spontaneously. Leaving tubing unsecured or tying a knot
Suction, gloves, masks, or other equipment increases the risk of urethral injury or
are important but do not replace the catheter-related complications.
immediate life-saving function of the Ambu
bag.

Q. A nursing officer is caring for a patient


with an indwelling urinary catheter.
MEDICAL SURGICAL NURSING Q. A 54-year-old man presents to the
Q. As a Nursing Officer, you are assessing emergency department with severe chest
three clients’ blood sugar reports: pain radiating to the left arm for the past 30
Client 1: HbA1c = 7.2% minutes. As a Nursing Officer, you know
Client 2: Fasting Blood Sugar = 122 mg/dl that the “golden period” for initiating
Client 3: 2-hour GTT = 131 mg/dl treatment in myocardial infarction (MI) is:
Based on diagnostic criteria, how will you
categorize them? 1. First 30 minutes after chest pain
2. First 1 hour after chest pain
1. Client 1 – Normal, Client 2 – Pre-diabetic, 3. First 3 hours after chest pain
Client 3 – Diabetic 4. First 6 hours after chest pain
2. Client 1 – Diabetic, Client 2 – Correct Answer: 2. First 1 hour after chest
Pre-diabetic, Client 3 – Normal pain
3. Client 1 – Diabetic, Client 2 – Normal,
Client 3 – Pre-diabetic Rationale (Point-wise):
4. Client 1 – Pre-diabetic, Client 2 – 1. Golden Hour in MI → First 1 hour after
Diabetic, Client 3 – Normal onset of chest pain.
Early recognition + treatment (aspirin,
Correct Answer: 2. Client 1 – Diabetic, oxygen, nitrates, morphine, reperfusion
Client 2 – Pre-diabetic, Client 3 – Normal therapy) can save ischemic myocardium.
2. Thrombolytic therapy (fibrinolysis):
Rationale (Detailed in Points): Most effective if started within 30 minutes
1. HbA1c (Glycated Hemoglobin): of hospital arrival (“door-to-needle time”).
Normal: < 5.7% 3. Primary PCI (angioplasty):
Pre-diabetic: 5.7 – 6.4% Best outcomes if done within 90 minutes of
Diabetic: ≥ 6.5% first medical contact (“door-to-balloon
Client 1 = 7.2% → Diabetic time”).
4. After the golden hour, myocardial
2. Fasting Blood Sugar (FBS): necrosis becomes irreversible, and
Normal: < 100 mg/dl mortality increases significantly
Pre-diabetic (Impaired Fasting Glucose):
100 – 125 mg/dl Key Exam Line:
Diabetic: ≥ 126 mg/dl Golden Period = First 1 hour in MI
Client 2 = 122 mg/dl → Pre-diabetic But reperfusion therapy should ideally be
started within 6 hours (the critical window).
3. Oral Glucose Tolerance Test (2-hour Q. A 35-year-old patient presents with mild
GTT): hyperglycemia and has a gradual onset of
Normal: < 140 mg/dl symptoms over several months.
Pre-diabetic (Impaired Glucose Tolerance): Autoantibody testing is positive, and the
140 – 199 mg/dl patient is not obese. He initially responds to
Diabetic: ≥ 200 mg/dl oral hypoglycemic agents but eventually
Client 3 = 131 mg/dl → Normal requires insulin. This type of diabetes is
best classified as:
1. Type 1 Diabetes Mellitus 1. Client 1 – Cataract, Client 2 – Retinal
2. Type 2 Diabetes Mellitus Detachment
3. Type 1.5 Diabetes (LADA) 2. Client 1 – Retinal Detachment, Client 2 –
4. Gestational Diabetes Mellitus Cataract
3. Client 1 – Glaucoma, Client 2 – Cataract
Correct Answer: 3. Type 1.5 Diabetes 4. Client 1 – Diabetic Retinopathy, Client 2 –
(LADA) Hyperglycemia-induced refractive changes

Rationale (Point-wise): Correct Answer: 2. Client 1 – Retinal


1. Definition: Detachment, Client 2 – Cataract
Type 1.5 Diabetes, also called LADA (Latent
Autoimmune Diabetes in Adults), is a form Rationale (Point-wise):
of diabetes with features of both Type 1 and 1. Client 1 – Complete blindness:
Type 2. Cause: Retinal detachment
Often occurs in advanced diabetic
2. Key Features: retinopathy due to traction from
Age of onset: usually >30 years fibrovascular tissue.
Gradual onset of hyperglycemia Leads to sudden or complete vision loss.
Autoantibody positive (GAD antibodies)
Initially responsive to oral hypoglycemics, 2. Client 2 – Blurring of vision:
eventually requires insulin Cause: Cataract
Usually non-obese Diabetes accelerates lens opacification,
leading to gradual blurring of vision.
3. Differentiation: Typically slowly progressive and may
Type 1 DM: sudden onset, usually in require surgical correction.
children/adolescents, insulin-dependent
from diagnosis 3. Other points:
Type 2 DM: insulin resistance, obesity Retinal detachment → ophthalmic
common, oral agents effective for a long emergency.
time Cataract → common in chronic diabetes but
Type 1.5/LADA: overlaps both types, slow less acute.
autoimmune destruction
Key Exam Tip:
4. Importance for Nursing Care: Sudden blindness in DM → Retinal
Monitor blood glucose trends, educate detachment (complication of proliferative
about insulin initiation, and screen for retinopathy)
autoimmune markers. Gradual blurring → Cataract

Q. Two clients with diabetes mellitus Q. A nursing student asks about the
present with visual complaints: mechanism of action of aldosterone.
Client 1: Sudden complete blindness Aldosterone primarily exerts its effects by
Client 2: Blurring of vision binding to which type of receptor?
What is the most likely cause in each case?
1. G-protein coupled receptor (GPCR) 1. Sulfonylurea
2. Intracellular/nuclear receptor 2. Metformin
3. Tyrosine kinase receptor 3. Semaglutide
4. Ligand-gated ion channel 4. Thiazolidinedione
Correct Answer: 2. Intracellular/nuclear Correct Answer: 3. Semaglutide
receptor
Rationale - Rationale (Point-wise):
1. Aldosterone: 1. Semaglutide:
A mineralocorticoid hormone produced by A GLP-1 receptor agonist
the adrenal cortex (zona glomerulosa). Stimulates glucose-dependent insulin
secretion
2. Mechanism of Action: Suppresses glucagon
Diffuses into target cells (principal cells of Slows gastric emptying, promoting satiety
renal collecting ducts). and weight loss
Binds to intracellular/nuclear
mineralocorticoid receptor (MR). 2. Advantages in Obese Type 2 DM:
The receptor-hormone complex Helps weight reduction while lowering
translocates to the nucleus and acts as a HbA1c
transcription factor, regulating gene Low risk of hypoglycemia
expression. Cardiovascular protective effects
demonstrated in trials
3. Effect:
Increases expression of Na⁺ channels 3. Other Drugs:
(ENaC) and Na⁺/K⁺ ATPase. Metformin: First-line, weight neutral or
Promotes sodium and water reabsorption modest weight loss
and potassium excretion. Sulfonylurea: Risk of weight gain and
4. Other receptors: hypoglycemia
GPCR → e.g., epinephrine, angiotensin II Thiazolidinedione: May cause weight gain
Tyrosine kinase receptor → e.g., insulin, and fluid retention
growth factors
Ligand-gated ion channel → e.g., nicotinic 4. Clinical Guideline Tip:
acetylcholine receptor In obese patients needing both glycemic
control + weight loss, GLP-1 agonists (e.g.,
Key Exam Tip: Semaglutide) are preferred.
Steroid hormones (aldosterone, cortisol,
estrogen, testosterone) → act via Key Exam Tip:
intracellular/nuclear receptors. Semaglutide = Type 2 DM + Obesity →
Q. A 50-year-old obese patient with newly Glycemic control + Weight loss
diagnosed Type 2 Diabetes Mellitus
requires initiation of therapy. Which of the Q. A 40-year-old patient with Type 2
following drugs is most appropriate for Diabetes Mellitus is receiving
glycemic control and weight reduction? subcutaneous insulin. The patient has a
healed LSCS incision scar on the abdomen. Q. A 35-year-old patient with Addison’s
As a Nursing Officer, which of the following disease has been taking continuous
is correct regarding insulin administration dexamethasone for several months. Which
at the scar site? nursing advice is most important for this
patient?
1. Insulin absorption is faster at scarred
sites, so dose should be reduced 1. Avoid crowded places and report any
2. Insulin absorption is slower or signs of infection immediately
unpredictable at scarred sites, so injections 2. Reduce fluid intake to prevent edema
should avoid scar tissue 3. Increase physical activity to improve
3. Scar tissue has no effect on insulin immunity
absorption, so injections can be given 4. Take an extra dose of dexamethasone if
anywhere fever occurs
4. Scar tissue increases risk of Correct Answer: 1. Avoid crowded places
hypoglycemia if insulin is injected there and report any signs of infection
immediately
Correct Answer: 2. Insulin absorption is
slower or unpredictable at scarred sites, so Rationale:
injections should avoid scar tissue Dexamethasone suppresses the immune
system, and a decreased TLC further
Rationale (Point-wise): increases susceptibility to infections.
1. Insulin Absorption Factors: Patients should avoid crowded places,
Absorption depends on subcutaneous practice good hygiene, and report fevers,
blood flow and tissue integrity. sore throat, or other infection signs
Scar tissue has fibrosis, decreased immediately.
vascularity, and altered subcutaneous fat → B, C, and D are incorrect because they do
slower or unpredictable absorption. not address the immediate risk of infection
and could worsen the patient’s condition.
2. Clinical Implications:
Injecting insulin into scarred areas can lead Q. A 32-year-old woman presents 3 months
to erratic glucose control (hyperglycemia or after a complicated postpartum hemorrhage
delayed action). with fatigue, inability to lactate, and
Nursing practice: rotate injection sites, amenorrhea. Laboratory investigations
avoid scars, lipohypertrophy, or inflamed reveal low cortisol, low TSH, and low
areas. prolactin levels. Which of the following is
the most likely diagnosis?
3. Other Options:
A) Absorption is not faster in scar tissue; 1. Addison’s disease
it’s slower 2. Sheehan syndrome
C) Scar tissue does affect absorption 3. Hypothyroidism
D) Hypoglycemia risk is not increased 4. Simmonds disease
directly, but delayed insulin can lead to
hyperglycemia Correct Answer: 2. Sheehan syndrome
Rationale: Hepatitis A and E → Spread via the
The patient’s history of postpartum fecal–oral route and are not transmitted
hemorrhage is key in differentiating perinatally.
Sheehan syndrome from other causes of
panhypopituitarism. Q. A 25-year-old patient presents with mild
Sheehan syndrome is ischemic necrosis of visual blurring. On slit-lamp examination,
the anterior pituitary after severe obstetric the anterior lens capsule shows a
blood loss, causing failure of lactation, characteristic sunflower-shaped opacity.
amenorrhea, fatigue, and hypotension. This ocular finding is most commonly
Simmonds disease refers to non-obstetric associated with:
causes of panhypopituitarism, such as
pituitary tumors, trauma, infections, 1. Wilson disease
autoimmune disease, or pituitary apoplexy. 2. Diabetes mellitus
In Simmonds disease, there is usually no 3. Hypertension
history of obstetric hemorrhage. 4. Marfan syndrome
Addison’s disease and primary
hypothyroidism can cause hormonal Correct Answer: 1. Wilson disease
deficiencies but do not cause
panhypopituitarism with failure of lactation. Rationale:
1. Definition: Sunflower cataract is an
Q. A nurse is counseling a pregnant woman anterior lens capsule opacity with a central
regarding the risk of transmitting hepatitis disc and radiating petal-like pattern.
to her newborn during childbirth. Which 2. Association: It is typically seen in Wilson
type of hepatitis has the highest risk of disease, a disorder of copper metabolism.
perinatal transmission? 3. Pathophysiology: Excess copper
deposition in the lens causes this
1. Hepatitis A distinctive appearance.
2. Hepatitis E 4. Other ocular sign: Kayser-Fleischer rings
3. Hepatitis C (copper deposition in Descemet’s
4. Hepatitis B membrane) often accompany it.
5. Exclusion of other options: Diabetes,
Correct Answer: 4. Hepatitis B hypertension, and Marfan syndrome do not
produce sunflower cataracts.
Rationale:
Hepatitis B → Highest priority; risk of
vertical transmission is very high,
especially if the mother is HBsAg & HBeAg
positive.
Hepatitis C → Can also be transmitted
during childbirth, but risk is much lower
than HBV.
Q. This clinical feature is most commonly 1. A comprehensive, head-to-toe
associated with which condition? assessment of all body systems
2. An in-depth assessment limited to the
patient’s airway only
3. An assessment directed at the specific
system related to the patient’s current
problem
4. A rapid assessment performed only in
life-threatening emergencies

Correct Answer: 3. An assessment directed


1. Cushing’s syndrome at the specific system related to the
2. Acromegaly patient’s current problem
3. Graves’ disease (thyrotoxicosis)
4. Myasthenia gravis Rationale:
Focused assessment is problem-oriented
Correct Answer: 3. Graves’ disease and performed to evaluate a specific
(thyrotoxicosis) complaint, symptom, or body system (e.g.,
respiratory system for shortness of breath).
Rationale: Comprehensive assessment = complete
Exophthalmos (proptosis) is the forward head-to-toe evaluation (baseline,
protrusion of the eyeball beyond the orbital admission).
margin. Emergency assessment = rapid evaluation
The most common cause is Graves’ in life-threatening conditions (e.g., trauma).
disease, an autoimmune thyroid disorder Focused helps in monitoring and
where antibodies stimulate orbital re-evaluating progress of a specific issue
fibroblasts → inflammation, edema, and fat during ongoing care.
deposition in the orbit.
It is a hallmark of thyroid eye disease, often Q. Identify the instrument.
associated with lid lag and staring look.
Other causes (rare in exam context): orbital
tumors, cavernous sinus thrombosis,
trauma.

Q. A nurse is caring for a 50-year-old


patient who has just returned from surgery
with complaints of shortness of breath and
chest discomfort. The nurse decides to
perform a focused assessment. Which of
the following best describes this type of 1. Lumbar puncture needle
assessment? 2. Core biopsy needle (Tru-cut needle)
3. Bone marrow aspiration needle
4. Fine needle aspiration (FNA) needle
Correct Answer: 2. Core biopsy needle McBurney’s point: Located at the junction
(Tru-cut needle) of the lateral one-third and medial
two-thirds of the line from the right anterior
Rationale: superior iliac spine (ASIS) to the umbilicus.
A core biopsy needle (often called Tru-cut It is the classical site of tenderness in acute
needle) is designed to remove a core appendicitis.
(cylindrical sample) of tissue. Lanz point: At the junction of the right third
It provides an intact tissue architecture, and middle third of the line joining both
making it superior to FNAC for ASIS – also used for appendicitis.
histopathology. Morris point: At the junction of the middle
Commonly used for: breast lumps, liver and outer third of the line from the
biopsy, renal biopsy, prostate biopsy. umbilicus to the right ASIS.
FNA needle → only aspirates cells, not a Sherren’s triangle: Area of hyperesthesia in
tissue core. appendicitis.
Bone marrow aspiration needle → used for
marrow sampling, not solid organ tissue. Q. A 30-year-old male is brought to the
Lumbar puncture needle → used for CSF emergency room after a motor vehicle
sampling. accident. On observation, the nurse notes
that during inspiration, a segment of the
Q. A 22-year-old male presents with acute patient’s chest wall moves inward instead
pain in the right lower abdomen. On of outward, while the rest of the chest
examination, maximum tenderness is noted expands normally. This finding is referred
at a point located one-third of the distance to as:
from the anterior superior iliac spine to the
umbilicus on the right side. This point is 1. Kussmaul breathing
known as: 2. Paradoxical respiration
3. Cheyne-Stokes respiration
1. Lanz point 4. Apneustic breathing
2. McBurney’s point
3. Morris point Correct Answer: 2. Paradoxical respiration
4. Sherren’s triangle
Rationale:
Correct Answer: 2. McBurney’s point Paradoxical respiration occurs when a
segment of the chest wall moves opposite
Rationale: to the normal movement during breathing.
Most commonly seen in flail chest, where
multiple rib fractures lead to an unstable
segment.
During inspiration, the negative
intrathoracic pressure pulls the flail
segment inward, and during expiration, it
bulges outward.
Other types of abnormal breathing:
Kussmaul breathing → deep, labored, in coughing and sneezing. As a nursing
metabolic acidosis. officer, this finding is best described as:
Cheyne-Stokes → cyclic waxing and waning
of tidal volume. 1. Referred pain
Apneustic → prolonged inspiration. 2. Radiating pain
3. Phantom pain
Q. A 28-year-old man is undergoing 4. Psychogenic pain
mechanical ventilation. The physician
explains that lung inflation inhibits further Correct Answer: 2. Radiating pain
inspiration to prevent overinflation of the
lungs. This protective mechanism is Rationale:
mediated by: Radiating pain → Pain starts at one site and
spreads along the course of a nerve root
1. Hering–Breuer reflex (e.g., sciatica).
2. Cough reflex Referred pain → Pain felt in an area distant
3. Bainbridge reflex from the site of origin but not along the
4. Valsalva reflex nerve pathway (e.g., left arm pain in MI).
Phantom pain → Felt in a
Correct Answer: 1. Hering–Breuer reflex missing/amputated limb.
Psychogenic pain → Pain without organic
Rationale: cause, linked to psychological factors.
The Hering–Breuer reflex is a pulmonary
stretch receptor reflex that prevents Q. The image shown is a diagnostic chart
overinflation of the lungs. used in ophthalmology. Identify its name :
Mechanism: Stretch receptors in the
bronchi and bronchioles send signals via
the vagus nerve to the medullary
respiratory centers → inhibiting inspiration.

Clinical relevance:
Protects the lungs from barotrauma.
Important in mechanical ventilation
settings.
Other reflexes:
Cough reflex → clears the airway.
Bainbridge reflex → responds to increased 1. Snellen chart.
atrial pressure. 2. Jaeger chart .
Valsalva reflex → affects heart rate and 3. Ishihara chart .
intrathoracic pressure. 4. Perimetry chart.

Q. A 48-year-old male presents with severe Correct Answer:3. Ishihara chart – Color
low back pain that extends down his left leg vision testing
up to the ankle. The pain increases on
Rationale: Q. The given X-ray shown below .This
The Ishihara chart consists of colored dot radiological feature is most commonly
patterns forming numbers/shapes. associated with which condition?
It is mainly used for detecting red-green
color blindness (most common type).
Snellen chart → distance vision,
Jaeger chart → near vision,
Perimetry → visual fields.

Q. The given chest X-ray shows . Which of


the following is the most likely diagnosis?

1. Pyloric stenosis
2. Duodenal atresia
3. Hirschsprung’s disease
4. Malrotation with volvulus

Correct Answer: 2. Duodenal atresia

Rationale:
Double bubble sign → two air-filled bubbles
1. Pleural effusion (stomach + proximal duodenum) with
2. Consolidation absence of distal bowel gas,
3. Pneumothorax pathognomonic for duodenal atresia.
4. Pulmonary edema Pyloric stenosis → shows dilated stomach
but no double bubble.
Correct Answer: 3. Pneumothorax Hirschsprung’s disease → massive colonic
dilatation, not double bubble.
Rationale: Malrotation with volvulus → may show
Pneumothorax → hyperlucent area with abnormal gas patterns but not the classical
absent lung markings, visible pleural line, double bubble.
and collapse of underlying lung.
Pleural effusion → homogenous opacity Q. A patient with Non-Hodgkin’s lymphoma
with meniscus sign. receiving chemotherapy develops nausea,
Consolidation → patchy or homogenous vomiting, arrhythmias, and decreased urine
opacity with air bronchogram. output. Laboratory results show
Pulmonary edema → bilateral perihilar “bat hyperkalemia, hyperphosphatemia,
wing” opacities. hypocalcemia, and hyperuricemia. Which of
the following is the most likely diagnosis?
1. Syndrome of Inappropriate ADH
Secretion (SIADH)
2. Tumor Lysis Syndrome Left lateral is useful in pregnancy or to
3. Hypercalcemia of malignancy prevent aspiration but not ideal here.
4. Hepatorenal Syndrome
Q. A nurse encounters an adult patient who
Correct Answer: 2. Tumor Lysis Syndrome has suddenly collapsed and is
unresponsive. According to basic life
Rationale: support (BLS) guidelines, what is the
TLS occurs due to rapid lysis of tumor cells correct sequence of steps to follow?
after chemotherapy/radiotherapy, releasing
intracellular contents. 1. Airway → Breathing → Circulation →
Lab findings: ↑ potassium, ↑ phosphate, ↑ Defibrillation
uric acid, ↓ calcium. 2. Circulation → Airway → Breathing →
Symptoms: nausea, vomiting, renal failure, Defibrillation
arrhythmias, seizures. 3. Check responsiveness → Call for help →
SIADH presents with hyponatremia, not this Airway → Breathing → Circulation →
combination. Defibrillation
Hypercalcemia of malignancy shows ↑ Ca 4. Defibrillation → Airway → Breathing →
(opposite of TLS). Circulation
Hepatorenal syndrome is associated with
liver disease and oliguria, not the Correct Answer: 3. Check responsiveness
electrolyte pattern here. → Call for help → Airway → Breathing →
Circulation → Defibrillation
Q. A patient is admitted with acute
pulmonary edema. The nurse knows that Rationale:
the best position to improve oxygenation The BLS/CPR sequence for adults follows:
and reduce preload on the heart is: 1. Check responsiveness and safety.
2. Call for help and activate emergency
1. Supine position with legs elevated response.
2. High Fowler’s position with legs 3. Airway: Ensure airway is open.
dependent 4. Breathing: Check for normal breathing.
3. Left lateral position 5. Circulation: Start chest compressions if
4. Trendelenburg position no pulse.
6. Defibrillation: Use AED as soon as
Correct Answer: 2. High Fowler’s position available.
with legs dependent This systematic approach maximizes
survival and follows international CPR
Rationale: guidelines.
High Fowler’s with legs dependent helps
decrease venous return (preload) to the Q. A 12-year-old child is admitted with a
heart, reduces pulmonary congestion, and sickle cell crisis, presenting with severe
improves lung expansion. pain, fever, and pallor. As a nursing officer,
Supine/Trendelenburg worsen venous which of the following actions is the least
return and pulmonary edema. priority?
1. Administer prescribed analgesics forPatients must take it with a full glass of
pain relief water on an empty stomach and stay
2. Encourage oral fluids and maintain upright for at least 30 minutes to prevent
hydration reflux and esophageal damage.
3. Apply cold packs to painful joints Taking it with milk/food or at bedtime
4. Monitor vital signs and oxygen saturation
increases risk of poor absorption or
esophageal injury.
Correct Answer: 3. Apply cold packs to Tablets should not be crushed unless
painful joints specifically formulated for that purpose.

Rationale: Q. A patient suddenly develops severe


Pain management, hydration, and dyspnea, hypotension, distended neck
monitoring are top priorities in a sickle cell veins, and tracheal deviation to the
crisis to prevent complications like opposite side after a chest trauma. The
vaso-occlusion and hypoxia. nurse suspects tension pneumothorax.
Cold therapy is contraindicated, as cold can Which of the following is the most
induce vasoconstriction, worsening appropriate immediate nursing action?
sickling and pain.
Nursing interventions must focus on 1. Prepare and assist in needle
warmth, hydration, oxygenation, and decompression or chest tube insertion
analgesia. 2. Administer high-dose antibiotics
3. Place the patient in a Trendelenburg
Q. A 60-year-old patient has been position
prescribed Alendronate for osteoporosis. 4. Apply oxygen via nasal cannula only
Which of the following nursing instructions
is most important to include in patient Correct Answer: 1. Prepare and assist in
education? needle decompression or chest tube
insertion
1. Take the tablet with a full glass of water
and remain upright for at least 30 minutes Rationale:
2. Take the tablet with milk or food to Tension pneumothorax is a life-threatening
reduce stomach upset emergency; immediate decompression is
3. It can be taken at bedtime before sleeping required to relieve pressure on the lungs
4. Crush the tablet if swallowing is difficult and heart.
High-dose antibiotics or Trendelenburg
Correct Answer: 1. Take the tablet with a full position are not relevant and may delay
glass of water and remain upright for at life-saving intervention.
least 30 minutes Oxygen supplementation alone is
insufficient; definitive decompression is
Rationale: critical.
Alendronate can cause esophageal Nursing role includes rapid recognition,
irritation or ulceration if not taken properly. calling for help, preparing equipment,
assisting the physician, and monitoring Q. Which type of suture is being used in
vital signs. this image?

Q. Identify the type of suturing used in this


image.

1. Simple interrupted suture


2. Horizontal mattress suture
3. Purse-string suture
4. Vertical mattress suture

Correct Answer: 3. Purse-string suture

Rationale:
1. Simple interrupted suture Purse-string suture is a circular, continuous
2. Continuous running suture suture used to close circular wounds,
3. Horizontal mattress suture around stomas, drains, or for delayed
4. Purse-string suture closure.
When the suture is pulled tight, it cinches
Correct Answer: 3. Horizontal mattress the tissue together like a purse, achieving
suture closure.
Simple interrupted and mattress sutures
Rationale: are linear and do not encircle the wound.
Horizontal mattress suture loops parallel to
the wound edge, providing tension relief Q. A patient develops antisocial behavior,
and eversion. lack of social inhibition, and personality
Simple interrupted: each stitch is changes after a head injury. Which area of
independent and perpendicular to wound the brain is most likely injured?
edge.
Continuous running: one long suture along 1. Occipital lobe
the wound edge. 2. Temporal lobe
Purse-string: circular closure, often for 3. Frontal lobe
ostomies or around drain sites. 4. Parietal lobe

Correct Answer: 3. Frontal lobe


Rationale: Q. A patient presents with clear fluid
The frontal lobe (especially prefrontal draining from the ear after a head injury.
cortex) is responsible for personality, The nurse suspects CSF otorrhea. Which
judgment, emotions, and social behavior. diagnostic test is MOST specific to confirm
Injury here can cause antisocial behavior, the presence of CSF?
impulsivity, and loss of inhibition.
Occipital lobe → vision. 1. CT scan of head
Temporal lobe → memory, hearing. 2. Beta-2 transferrin test
Parietal lobe → sensory integration. 3. Glucose test of ear discharge
4. MRI brain
Q. Match the following clinical terms with
their correct description: Correct Answer: 2. Beta-2 transferrin test
Column A (Disorders)
i. Abulia Rationale:
ii. Aphasia Beta-2 transferrin test → gold standard,
iii. Apraxia highly specific for CSF.
iv. Apathy Glucose test → not reliable, because
Column B (Features) nasal/ear secretions may also contain
a. Inability to perform learned motor tasks glucose → false positive.
despite normal motor strength CT/MRI → can detect fracture or site of leak
b. Lack of drive, initiative, or motivation but not confirm if fluid is CSF.
c. Impairment of language production or
comprehension Q. A patient with severe head injury and
d. Decreased interest, motivation, and active intracranial bleed is prescribed IV
self-care Mannitol by the doctor. As a Nursing
Options: Officer, what should be your immediate
action?
1. i-b, ii-c, iii-a, iv-d
2. i-d, ii-b, iii-c, iv-a 1. Administer Mannitol as ordered to reduce
3. i-a, ii-d, iii-b, iv-c ICP
4. i-c, ii-a, iii-d, iv-b 2. Question the order, as Mannitol is
contraindicated in active bleeding
Correct Answer: 1. i-b, ii-c, iii-a, iv-d 3. Place the patient in Trendelenburg
position
Rationale: 4. Restrict oral fluids to prevent cerebral
Abulia → b (lack of drive/motivation) edema
Aphasia → c (language impairment)
Apraxia → a (inability to perform learned Correct Answer: 2. Question the order, as
movements despite intact motor function) Mannitol is contraindicated in active
Apathy → d (decreased self-care, lack of bleeding
interest)
Rationale:
Mannitol is an osmotic diuretic → used to Correct Answer: 3. Group B Streptococcus
lower ICP.
However, in active intracranial hemorrhage, Rationale:
it is generally contraindicated, because: Neonatal meningitis (<28 days of life):
It increases plasma volume → may worsen Most common cause: Group B
bleeding. Streptococcus (GBS)
It can cause rebound increase in ICP. Other causes: E. coli, Listeria
The nurse’s role is to ensure safe monocytogenes
medication administration → questioning Streptococcus pneumoniae → more
the order in this scenario is correct. common in infants >1 month and children.
Neisseria meningitidis → common in older
Q. A patient presents with complete children/adolescents.
paralysis of both upper limbs, both lower Haemophilus influenzae type b → incidence
limbs, and trunk muscles following a high decreased due to Hib vaccination.
cervical spinal cord injury. This condition is
referred to as: Q. A patient develops post-dural puncture
headache (PDPH) after a lumbar puncture.
1. Quadriplegia The drug of choice (DOC) for relieving this
2. Pentoplegia headache is:
3. Paraplegia
4. Hemiplegia 1. Acetaminophen
2. Caffeine
Correct Answer: 2. Pentoplegia 3. Sumatriptan
4. Mannitol
Rationale:
Pentoplegia → paralysis of all four limbs + Correct Answer: 2. Caffeine
trunk.
Quadriplegia → paralysis of all four limbs Rationale:
only. Post-dural puncture headache is due to
Paraplegia → paralysis of both lower limbs. CSF leakage → low CSF pressure, causing
Hemiplegia → paralysis of one side of the positional headache.
body. Caffeine is the drug of choice because:
It causes cerebral vasoconstriction →
Q. A 10-day-old neonate presents with reduces headache.
fever, irritability, poor feeding, and bulging Can be given orally or IV.
fontanelle. The most common cause of Acetaminophen/NSAIDs → supportive, mild
neonatal meningitis is: relief only.
Sumatriptan → used for migraine, not
1. Streptococcus pneumoniae PDPH.
2. Neisseria meningitidis Mannitol → used to reduce raised ICP, not
3. Group B Streptococcus low CSF pressure headache.
4. Haemophilus influenzae type b
Q. During lumbar puncture in a patient with Correct Answer: 2. Ensure a clutter-free
suspected meningitis, how much environment and use side rails as needed
cerebrospinal fluid (CSF) should be
collected, and how many vials are typically Rationale:
required for standard laboratory analysis? Huntington’s chorea → involuntary
movements, impaired coordination → high
1. 3–5 mL, 2 vials risk of falls.
2. 10–15 mL, 3 vials Nursing precautions:
3. upto 20 mL, 4 vials Maintain clutter-free floors
4.upto 25 mL, 5 vials Use side rails and non-slip footwear
Provide assistance during ambulation
Correct Answer: 3. 20 mL, 4 vials Keep bed at safe height (low, not high)
Avoid unnecessary restraints or complete
Rationale: confinement → maintain mobility and
CSF collection: Adults → up to 20 mL can independence
be safely collected. Option A (high bed) increases risk of fall.
Vials: Option C (unassisted walking) is unsafe.
1. Chemistry – glucose, protein Option D (restrict all activity) leads to
2. Microbiology – Gram stain, culture deconditioning.
3. Hematology – total and differential cell
count Q. A nurse is educating the community
4. Additional tests – PCR, antigen detection, about early recognition of stroke. Which of
or backup the following is the correct BEFAST
Collecting in 4 separate vials ensures acronym?
adequate sample for all required
investigations. 1. Balance, Eyes, Face, Arms, Speech, Time
Sequential collection avoids contamination 2. Breathing, Eyes, Face, Arms, Speech,
between tests. Temperature
3. Balance, Eyes, Fingers, Arms, Speech,
Q. A patient with Huntington’s chorea has Time
severe involuntary movements, poor 4. Breathing, Eyes, Face, Abdomen,
balance, and impaired coordination. Which Speech, Temperature
of the following is the most important
nursing precaution to prevent falls? Correct Answer: 1. Balance, Eyes, Face,
Arms, Speech, Time
1. Keep the bed in high position and use
soft pillows Rationale:
2. Ensure a clutter-free environment and BEFAST is a mnemonic to identify early
use side rails as needed signs of stroke:
3. Encourage patient to walk unassisted to 1. B – Balance: Sudden loss of balance or
improve mobility coordination
4. Restrict all physical activity and keep 2. E – Eyes: Sudden vision changes
patient confined to bed 3. F – Face: Facial droop on one side
4. A – Arms: Weakness in one arm or leg COMMUNITY HEALTH NURSING
5. S – Speech: Slurred or confused speech Q. A country is experiencing a higher
6. T – Time: Immediate action; call proportion of older population, very low
emergency services death rate, high dependency ratio, and has
Early recognition improves time-sensitive entered Stage V of the demographic cycle.
interventions like thrombolysis. Which of the following best describes this
situation?
Q. A 25-year-old patient complains of blurry
vision both at a distance and close up. On 1. Demographic dividend
examination, the ophthalmologist finds that 2. Demographic liability
the patient requires corrective lenses. 3. Population explosion
Which of the following statements about 4. Demographic transition
refractive errors is correct?
Correct Answer: 2. Demographic liability
1. They are caused by clouding of the lens
2. They result from improper focusing of Rationale:
light on the retina due to abnormalities in Demographic liability refers to the condition
the cornea or lens where an ageing population, low death rate,
3. They are always associated with and high dependency ratio create economic
increased intraocular pressure and social challenges.
4. They are irreversible and cannot be Demographic dividend is the
corrected with lenses opposite—when the working-age
population is high and dependency ratio is
Correct Answer: 2. They result from low.
improper focusing of light on the retina due Population explosion is characterized by
to abnormalities in the cornea or lens high birth rate and declining death rate
(earlier stage of demographic cycle).
Rationale: Demographic transition is the general
Refractive errors (myopia, hyperopia, process, but Stage V specifically refers to
astigmatism, presbyopia) occur when light declining birth rates and ageing population.
does not focus correctly on the retina, often
due to: Q. If the crude birth rate (CBR) of a country
Abnormal corneal curvature is 26 per 1000 population and the crude
Lens shape issues death rate (CDR) is 8 per 1000 population,
Axial length of the eyeball what will be the annual growth rate (AGR) of
Cataract (A) is lens clouding. the population?
Glaucoma (C) involves increased
intraocular pressure. 1. 1.8%
Most refractive errors are correctable with 2. 2.6%
glasses, contact lenses, or surgery (D is 3. 3.4%
false). 4. 8.0%

Correct Answer: 1. 1.8%


Rationale: 2. Offering comprehensive health
Formula: Annual Growth Rate (AGR) = (CBR screenings for women, including NCDs and
– CDR) ÷ 10 reproductive health issues
Here, (26 – 8) ÷ 10 = 18 ÷ 10 = 1.8% 3. Distributing free sanitary napkins to
This represents the natural growth of adolescent girls in rural areas
population per year (excluding migration). 4. Promoting only maternal immunisation
services
Q. Which of the following methods is
classified as a Long Acting Reversible Correct Answer: 2. Offering comprehensive
Contraceptive (LARC)? health screenings for women, including
NCDs and reproductive health issues
1. Combined oral contraceptive pills
2. Depot medroxyprogesterone acetate Rationale:
(DMPA) injection SNSPA focuses on women’s health and
3. Copper intrauterine device (Cu-IUD) empowerment, with special attention to
4. Male condom maternal and child care.
Screenings include hypertension, anaemia,
Correct Answer: 3. Copper intrauterine reproductive health, breast & cervical
device (Cu-IUD) cancer, TB, diabetes, dermatological
concerns, and sickle cell disease.
Rationale: The programme addresses gaps identified
LARC methods provide effective in national health surveys and provides
contraception for an extended period specialised counselling in tribal areas.
without requiring user action on a daily or A multidisciplinary team including
per-coital basis. gynaecologists, surgeons, and dentists
Examples: Copper IUD, ensures diagnosis, treatment, and
Levonorgestrel-releasing IUD, follow-up.
Contraceptive implants.
DMPA injection is long-acting but not Q. You are shown the following logo during
considered LARC as it requires reinjection a health awareness camp:
every 3 months.
Oral contraceptive pills and condoms are
short-acting user-dependent methods.

Q. The Swasth Nari Sashakt Parivar


Abhiyaan (SNSPA) was launched under the
Ministry of Health and Family Welfare with
the primary aim of:

1. Providing financial assistance for


institutional deliveries Identify the scheme ?

1. Janani Suraksha Yojana (JSY)


2. Rashtriya Bal Swasthya Karyakram Correct Answer: 2. Monitoring cervical
(RBSK) mucus changes during the menstrual cycle
3. Swasth Nari Sashakt Parivar Abhiyaan
(SNSPA) Rationale:
4. Pradhan Mantri Surakshit Matritva Billing Method (Ovulation Method): A
Abhiyan (PMSMA) natural family planning technique where
women monitor cervical mucus.
Correct Answer: 3. Swasth Nari Sashakt Fertile mucus = clear, stretchy, slippery (like
Parivar Abhiyaan (SNSPA) raw egg white) → indicates ovulation.
Couples avoid intercourse during fertile
Rationale: mucus days if preventing pregnancy.
The SNSPA logo is used to symbolise Basal body temperature method (A) is a
comprehensive women’s health screenings separate technique.
and empowerment. Calendar/Rhythm method (C) depends on
It covers NCDs, reproductive health, previous cycle length.
maternal care, and tribal health counselling.
Progesterone level testing (D) is diagnostic,
Other options (JSY, RBSK, PMSMA) are not practical for natural family planning.
separate schemes with different focus
areas. Q. Which of the following statements about
injectable contraceptives is correct?

1. NET-EN is given as 150 mcg IM every 3


months
2. NET-EN is given as 200 mcg IM every 2
months
3. NET-EN is given as 200 mg IM every 2
months
4. Both NET-EN and DMPA contain estrogen
+ progesterone

Correct Answer: 3. NET-EN is given as 200


Q. The Billing Method of contraception is mg IM every 2 months
based on which of the following
observations? Rationale:
NET-EN: Norethisterone enanthate, 200 mg
1. Recording basal body temperature daily IM every 2 months.
2. Monitoring cervical mucus changes DMPA: Depot Medroxyprogesterone
during the menstrual cycle acetate, 150 mg IM every 3 months.
3. Calculating fertile days by calendar Neither contains estrogen → both are
rhythm progestin-only injectables.
4. Measuring serum progesterone levels Options A and B are swapped doses to
create confusion.
Option D is incorrect because they are not 2. Normal finding, not clinically significant
combined injections. 3. Indicates past immunization with typhoid
vaccine
Q. A community health nurse is conducting 4. Suggestive of malaria infection
a health education program on smoking
cessation and vaccination drives to prevent Correct Answer: 1. Suggestive of typhoid
disease occurrence in healthy individuals. fever (significant titer)
These activities are examples of which level
of prevention? Rationale:
Widal test detects antibodies (agglutinins)
1. Primordial prevention against Salmonella typhi.
2. Primary prevention Significant titer in endemic areas is usually
3. Secondary prevention ≥ 1:160 for O antigen and/or ≥ 1:320 for H
4. Tertiary prevention antigen.
Rising titers on paired samples taken 7–10
Correct Answer: 2. Primary prevention days apart confirm diagnosis.
Option B is incorrect since the values are
Rationale: above the cutoff.
Primary prevention: Measures taken before Option C is not applicable (vaccine mainly
the onset of disease to prevent its raises H antibody, but O at this level
occurrence (e.g., immunization, health indicates infection).
education, lifestyle modification, use of seat Option D is unrelated.
belts).
Primordial prevention: Preventing the Q. What is the full form of UNDP?
emergence of risk factors themselves (e.g.,
discouraging junk food habits in children to 1. United Nations Development Program
prevent obesity). 2. United Nations Department of Policy
Secondary prevention: Early detection and 3. Universal Nations Development Plan
prompt treatment (e.g., screening tests like 4. United Nations Disaster Prevention
Pap smear, mammography).
Tertiary prevention: Limiting disability and Correct Answer: 1. United Nations
rehabilitation (e.g., physiotherapy in stroke Development Program
patients).
Rationale:
Q. A 25-year-old male presents with fever UNDP is the global development network of
for 7 days, abdominal discomfort, and the United Nations, focusing on poverty
relative bradycardia. His Widal test shows O reduction, sustainable development, and
agglutinin 1:160 and H agglutinin 1:320. As governance.
a nursing officer, how should this result be Other options are incorrect expansions and
interpreted? do not represent the UN organization.

1. Suggestive of typhoid fever (significant


titer)
Q. Which of the following groups are the Correct Answer: 3. Study that collects data
beneficiaries of the Integrated Child at a single point in time to assess
Development Services (ICDS) scheme? prevalence of disease or condition

1. Children below 6 years, adolescent girls, Rationale:


pregnant & lactating women Cross-sectional studies are observational
2. Children 6–14 years, only adolescent and measure exposure and outcome
girls simultaneously at a single point in time,
3. Children below 6 years, only pregnant making them useful for estimating
women prevalence but not for establishing
4. Only malnourished children and lactating causality.
mothers Option A = Cohort study,
Option B = Case-control study,
Correct Answer: 1. Children below 6 years, Option D = Randomized controlled trial
adolescent girls, pregnant & lactating (RCT).
women
Q. Which of the following statements about
Rationale: the cholera vaccine is correct?
ICDS Beneficiaries:
1. Children below 6 years → supplementary 1. It provides lifelong immunity after a
nutrition, preschool education, single dose
immunization. 2. Oral cholera vaccine is given as two
2. Adolescent girls (11–18 years) → doses at least 2 weeks apart
nutrition & health education (under SABLA 3. Injectable cholera vaccine is preferred for
scheme). mass immunization programs
3. Pregnant women → antenatal care, 4. Cholera vaccine is contraindicated in
nutrition, health education. children above 1 year of age
4. Lactating mothers → postnatal care,
nutrition support, counseling. Correct Answer: 2. Oral cholera vaccine is
given as two doses at least 2 weeks apart
Q. Which of the following best describes a
cross-sectional study design? Rationale:
Oral cholera vaccines (OCVs) are preferred
1. Study that follows a group of individuals for preventive use in endemic areas and
over time to assess incidence of disease outbreaks.
2. Study that compares patients with a The primary series is 2 doses given 2
disease to those without disease to find risk weeks apart (for persons ≥1 year).
factors Immunity lasts around 3 years, not lifelong.
3. Study that collects data at a single point Injectable vaccine is no longer
in time to assess prevalence of disease or recommended.
condition It is safe in children above 1 year of age.
4. Study where participants are randomly
assigned to intervention and control groups
Q. Which of the following is the correct 3. Repeat Mantoux test after 1 month
guideline regarding administration of 4. Give a second dose of BCG vaccine
multiple vaccines in children?
Correct Answer: 2. Consider this a normal
1. Two live parenteral vaccines must always response after BCG vaccination, no
be given at least 4 weeks apart if not treatment required
administered on the same day
2. Live oral and live parenteral vaccines Rationale:
should never be given together After BCG vaccination, Mantoux test may
3. Injectable vaccines should always be remain false-positive for several years due
administered in the same limb for better to cross-reactivity.
absorption A positive Mantoux in this context does not
4. Multiple vaccines cannot be given in the indicate active TB.
same visit due to risk of adverse events ATT is not started unless there are
symptoms or strong evidence of TB
Correct Answer: 1. Two live parenteral infection.
vaccines must always be given at least 4 Repeating Mantoux or revaccinating with
weeks apart if not administered on the BCG is not recommended.
same day
Q. Identify the program from the given logo:
Rationale:
Multiple vaccines can be administered
safely during the same visit.
Live parenteral vaccines (e.g., MMR,
varicella, yellow fever) can be given
together; if not, a minimum interval of 4
weeks must be maintained.
Live oral vaccines (e.g., OPV, rotavirus) can
be given with other vaccines without
spacing.
Different injectable vaccines should be
given in different sites/limbs, not the same
one. 1. National Tuberculosis Elimination
Programme (NTEP)
Q. A 6-month-old infant received BCG 2. National Leprosy Eradication Programme
vaccination 8 weeks ago. On Mantoux (NLEP)
testing, the induration is positive. What 3. National AIDS Control Programme
should be the next step? (NACP)
4. National Vector Borne Disease Control
1. Start full course of anti-tubercular Programme (NVBDCP)
treatment (ATT) immediately
2. Consider this a normal response after Correct Answer: 2. National Leprosy
BCG vaccination, no treatment required Eradication Programme (NLEP)
Rationale: Q. In a research study, which of the
The NLEP logo shows two protective hands following best defines a cross-sectional
around the Ashoka Chakra, representing study?
the nation’s effort to eliminate leprosy and
protect affected individuals. 1. A study that follows a group of subjects
It signifies care, rehabilitation, and over time to observe outcomes
prevention of stigma against leprosy 2. A study that collects data at a single
patients. point in time to assess prevalence of a
Other options are incorrect because: condition
NTEP uses a red circle with TB-related text. 3. A study that randomly assigns
NACP has a red ribbon symbol. participants to experimental and control
NVBDCP has different vector-borne disease groups
icons. 4. A study that reviews previously
published literature on a topic
Q. “Vasectomy Fortnight” is observed every
year in India with the aim of: Correct Answer: 2. A study that collects
data at a single point in time to assess
1. Promoting temporary methods of prevalence of a condition
contraception
2. Increasing acceptance of male Rationale:
sterilization Cross-sectional studies measure exposure
3. Screening males for infertility and outcome simultaneously at one point in
4. Providing awareness about sexually time; they are useful for assessing
transmitted diseases prevalence.
Cohort studies (A) follow subjects over
Correct Answer: 2. Increasing acceptance time.
of male sterilization Randomized controlled trials (C) involve
experimental assignment.
Rationale: Systematic reviews (D) analyze existing
Vasectomy Fortnight is observed annually literature.
(usually in November) under the National
Family Welfare Programme.
The goal is to remove myths and
misconceptions about vasectomy,
encourage men’s participation in family
planning, and promote non-scalpel
vasectomy (NSV) as a safe, permanent, and
outpatient procedure.
Other options are incorrect:
(a) focuses on temporary methods
(condoms, pills).
(c) is not the objective.
(d) relates to STI awareness, not vasectomy.
PEDIATRICS Rationale:
Q. Tanner’s Chart is used to assess which Metopic suture → Trigonocephaly
of the following in adolescents? (triangular forehead)
Coronal suture → Brachycephaly
1. Body mass index (BMI) (shortened AP diameter, wide head)
2. Sexual maturity and secondary sexual Lambdoid suture → Plagiocephaly
characteristics (posterior flattening, asymmetry)
3. Bone age using X-ray Sagittal suture → Dolichocephaly (long
4. Cognitive development narrow head)
Option B is correct because only C
Correct Answer: 2. Sexual maturity and (Lambdoid → Plagiocephaly) matches
secondary sexual characteristics exactly as written in the question stem.

Rationale: Q. A newborn presents with micrognathia,


Tanner’s Chart (also called Sexual Maturity clenched fists with overlapping fingers,
Rating, SMR) classifies pubertal rocker-bottom feet, and low-set ears. . This
development into 5 stages based on: condition is known as:
Breast development (thelarche) in girls
Genital development in boys 1. Down syndrome
Pubic and axillary hair growth in both sexes 2. Patau syndrome
It helps track normal puberty, identify 3. Edward syndrome
precocious or delayed puberty, and guide 4. Turner syndrome
endocrine evaluation.
A (BMI) measures body weight status. Correct Answer: 3. Edward syndrome
C (Bone age) is assessed via wrist X-ray
(Greulich-Pyle method). Rationale:
D (Cognitive development) is unrelated to Edward syndrome (Trisomy 18):
Tanner staging. Extra chromosome 18.
Key features:
Q. Match the cranial suture involved with Micrognathia (small jaw)
the corresponding skull deformity: Clenched fists with overlapping fingers
A) Metopic suture → Brachycephaly Rocker-bottom feet
B) Coronal suture → Dolichocephaly Low-set ears
C) Lambdoid suture → Plagiocephaly Growth retardation, heart defects (VSD,
D) Sagittal suture → Trigonocephaly ASD)
Which of the following is correct?

1. A and B
2. C only
3. C and D
4. None of the above

Correct Answer: 2. C only


Down syndrome (Trisomy 21): Flat face, Q. Match the following developmental
upslanting palpebral fissures, single palmar milestones with the approximate age of
crease. achievement:
Patau syndrome (Trisomy 13): A. Social smile
Holoprosencephaly, cleft lip/palate, B. Mouthing objects
polydactyly. C. Standing without support
Turner syndrome (45,X): Short stature, D. Running
webbed neck, amenorrhea (females only). E. Dressing/undressing
Options:
Q. A 2-week-old neonate presents with
jaundice, vomiting, poor feeding, and 1. B months, E months, 12 months, 18
hepatomegaly. On ophthalmic examination, months, 3 years
cataracts are noted. Laboratory tests reveal 2. E months, B months, 12 months, 18
reducing sugars in urine. The most likely months, 3 years
underlying metabolic disorder is: 3. B months, E months, 18 months, 12
months, 3 years
1. Galactosemia 4. B months, E months, 12 months, 3 years,
2. Diabetes mellitus 18 months
3. Wilson disease
4. Hypothyroidism Correct Answer: 1. 2 months, 5 months, 12
months, 18 months, 3 years
Correct Answer: 1. Galactosemia
Rationale (points):
Rationale (in points): 1. Social smile: First social smile appears at
1. Definition: Galactosemia is an inherited around 2 months.
disorder of galactose metabolism, usually 2. Mouthing objects: Babies start mouthing
due to galactose-1-phosphate things at around 5 months as part of
uridyltransferase (GALT) deficiency. sensory exploration.
2. Neonatal features: Jaundice, vomiting, 3. Standing without support: Achieved at
poor feeding, hepatomegaly, failure to around 12 months.
thrive. 4. Running: Most children begin running at
3. Ocular sign: Cataracts develop due to around 18 months.
accumulation of galactitol in the lens. 5. Dressing/undressing: Simple dressing or
4. Laboratory findings: Reducing sugars in undressing skills develop by 3 years.
urine; elevated galactose-1-phosphate in
blood. Q. Regarding human breast milk, which of
5. Exclusion of other options: Diabetes, the following statements is INCORRECT?
Wilson disease, and hypothyroidism do not
cause neonatal cataracts with reducing 1. Colostrum is richer in protein and
sugars in urine. antibodies than mature milk.
2. Breast milk is rich in vitamins A and C
but low in vitamins D and K.
3. It contains enough vitamin B12 for all Rationale :
exclusively breastfed infants regardless of 1. Foremilk vs Hindmilk: Foremilk is watery,
maternal diet. rich in lactose, but low in fat; hindmilk is
4. Lactose is the main carbohydrate, aiding high in fat and calories, helping the baby
calcium absorption and gut flora. feel full.
2. Frequent crying: Often occurs if the baby
Correct Answer: 3. It contains enough only consumes foremilk—leads to quick
vitamin B12 for all exclusively breastfed satiety but not lasting fullness.
infants regardless of maternal diet. 3. Health education: Advise the mother to
allow the baby to finish one breast
Rationale - completely before offering the second; this
1. Colostrum: True—high in protein and ensures adequate fat intake from hindmilk.
immunoglobulins. 4. Other options:
2. Vitamins: True—A and C are abundant; D A: Formula supplementation is not needed
and K are low. if breastfeeding technique is corrected.
3. Vitamin B12: Incorrect—B12 content C: Artificially feeding hindmilk is
depends on maternal stores; deficiency can unnecessary and impractical.
occur in vegetarian mothers. D: Reducing feeding frequency can worsen
4. Lactose: True—main carbohydrate, hunger and lead to poor weight gain.
promotes calcium absorption and healthy
gut microbiome. Q. A mother reports that her 3-week-old
infant refuses to breastfeed from the breast
Q. A mother reports that her 2-month-old and only accepts a bottle. She states that
infant cries frequently 10–15 minutes after she started bottle-feeding a few days ago
breastfeeding. She says the baby seems because the baby was fussy at the breast.
satisfied initially but wants to feed again This situation is most likely due to:
soon. On assessment, you find that the
baby mostly drinks foremilk and not 1. Mastitis
enough hindmilk. As a nurse, what is the 2. Nipple confusion
most appropriate health education to the 3. Tongue-tie
mother? 4. Poor let-down reflex

1. Give formula after every breastfeeding Correct Answer: 2. Nipple confusion


session
2. Encourage the baby to empty one breastRationale :
completely before switching to the other 1. Definition: Nipple confusion occurs when
3. Feed only hindmilk using a syringe a baby switches from breast to bottle and
4. Reduce the frequency of breastfeeding has difficulty latching correctly on the
breast.
Correct Answer: 2. Encourage the baby to 2. Cause: Often due to early introduction of
empty one breast completely before artificial nipples, especially before
switching breastfeeding is well-established.
3. Clinical features: Baby may refuse the 3. Route: IM preferred over oral for reliable
breast, show fussiness, or prefer bottle flow absorption, especially in high-risk
over breast flow. newborns.
4. Management/health education: 4. Purpose: Prevent early, classic, and late
Avoid early bottle or pacifier use until Vitamin K deficiency bleeding (VKDB).
breastfeeding is established (usually first 5. Clinical relevance: Proper dosing is
3–4 weeks). critical to reduce morbidity and mortality in
Encourage frequent, skin-to-skin vulnerable neonates.
breastfeeding and proper latch technique.
5. Exclusion of other options: Q. A child requires Vitamin A
Mastitis: Presents with breast pain, supplementation according to age. What is
redness, fever. the recommended dose for the following
Tongue-tie: May cause difficulty latching ages?
but not preference for bottle over breast. A. 3 months
Poor let-down reflex: Usually causes slow B. 9 months
milk flow, not outright refusal after bottle C. 1.5 years
use.
1. 50,000 IU at 3 months; 100,000 IU at 9
Q. A newborn is at high risk for vitamin K months; 200,000 IU at 1.5 years
deficiency due to underlying conditions. 2. 100,000 IU at 3 months; 100,000 IU at 9
The recommended intramuscular dose of months; 200,000 IU at 1.5 years
Vitamin K for the following conditions is: 3. 50,000 IU at 3 months; 50,000 IU at 9
months; 100,000 IU at 1.5 years
A. Severe acute malnutrition (SAM) 4. 100,000 IU at 3 months; 200,000 IU at 9
B. Hemolytic disease of the newborn (HDN) months; 200,000 IU at 1.5 years

1. 1 mg IM for both SAM and HDN Correct Answer: 1. 50,000 IU at 3 months;


2. 2.5 mg IM for SAM; 5 mg IM for HDN 100,000 IU at 9 months; 200,000 IU at 1.5
3. 0.5 mg IM for SAM; 1 mg IM for HDN years
4. 5 mg IM for SAM; 2.5 mg IM for HDN
Rationale :
Correct Answer: 2. 2.5 mg IM for SAM; 5 mg 1. Vitamin A supplementation: Used to
IM for HDN prevent xerophthalmia and vitamin A
deficiency in children at risk.
Rationale (points): 2. Recommended doses:
1. Standard dose: Healthy term newborns Infants 6–11 months: 100,000 IU every 4–6
receive 1 mg IM at birth. months (so at 9 months: 100,000 IU).
2. High-risk infants: Infants 12–59 months: 200,000 IU every 4–6
SAM: Higher risk of bleeding; 2.5 mg IM months (so at 1.5 years: 200,000 IU).
recommended. For younger infants (<6 months): If
HDN: Risk of coagulopathy due to indicated, 50,000 IU can be given.
hemolysis; 5 mg IM recommended. 3. Rationale: Dose increases with age to
meet nutritional requirements.
4. Route: Oral drops are commonly used; murmur is innocent (physiological) rather
safe and effective. than pathological?

Q. A nursing officer in a pediatric ward is 1. Harsh, holosystolic murmur with thrill


assigned to 4 newborns. Their vital signs 2. Soft, systolic murmur, grade 1–2, at the
are: left lower sternal border
A. Newborn A: HR 130 bpm, RR 36/min, BP 3. Associated cyanosis or tachypnea
70/40 mmHg, alert 4. Presence of abnormal heart sounds (S3,
B. Newborn B: HR 128 bpm, RR 38/min, BP S4)
72/42 mmHg, active
C. Newborn C: HR 135 bpm, RR 35/min, BP Correct Answer: 2. Soft, systolic murmur,
75/45 mmHg, feeding well grade 1–2, at the left lower sternal border
D. Newborn D: HR 150 bpm, RR 60/min, BP
62/41 mmHg, lethargic Rationale -
1. Nadas’ criteria help differentiate innocent
Which newborn should the nursing officer vs pathological murmurs in neonates.
assess first? 2. Innocent murmur characteristics:
1. Newborn A Soft (grade 1–2)
2. Newborn B Systolic
3. Newborn C Localized, usually at left lower sternal
4. Newborn D border
No associated cyanosis, thrill, or abnormal
Correct Answer: 4. Newborn D heart sounds
3. Pathological murmurs:
Rationale : Harsh, holosystolic
1. Priority: Nursing care follows the ABC Associated cyanosis, tachypnea, abnormal
(Airway, Breathing, Circulation) and triage heart sounds
principles. May have palpable thrill or abnormal pulses
2. Newborn D: Has abnormal vitals: 4. Clinical relevance: Using Nadas’ criteria
hypotension, tachycardia, tachypnea, and reduces unnecessary investigations in
lethargy—signs of shock or critical illness. healthy neonates.
Requires immediate attention.
3. Other newborns (A, B, C): Vitals are
within normal limits for age; stable and can
be assessed after the critical newborn.
4. Nursing principle: Always assess
critically unstable patients first, even if
assigned multiple stable patients.

Q. A 2-week-old infant is found to have a


heart murmur on routine examination.
According to Nadas’ criteria, which of the
following features suggests that the
Q. A 3-month-old infant presents with poor 2. Desmopressin
feeding and mild tachypnea. On 3. Oxybutynin
cardiovascular examination, a continuous 4. Furosemide
“machinery” murmur is heard at the upper
left sternal border. This murmur is most Correct Answer: 2. Desmopressin
characteristic of:
Rationale:
1. Atrial septal defect (ASD) Desmopressin (synthetic ADH) reduces
2. Ventricular septal defect (VSD) nocturnal urine production and is the
3. Patent ductus arteriosus (PDA) first-line pharmacological treatment for
4. Tetralogy of Fallot (TOF) primary nocturnal enuresis.
Imipramine is a second-line option if
Correct Answer: 3. Patent ductus arteriosus behavioral therapy fails; it has a higher risk
(PDA) of side effects.
Oxybutynin is used in overactive bladder,
Rationale : not primary enuresis.
1. Continuous machinery murmur: Heard Furosemide is a diuretic and would worsen
throughout systole and diastole, often bedwetting.
described as “machinery-like” or
“to-and-fro.” Q. A healthy full-term newborn requires
2. Location: Typically upper left sternal adequate nutrition for growth and
border, sometimes radiating to the back or development. What is the approximate daily
axilla. caloric requirement for a full-term
3. Associated features: Bounding pulses, newborn?
wide pulse pressure, possible signs of
heart failure in infants. 1. 50–60 kcal/kg/day
4. Exclusion of other defects: 2. 80–100 kcal/kg/day
ASD: Usually a systolic ejection murmur at 3. 110–120 kcal/kg/day
upper left sternal border, no continuous 4. 150–180 kcal/kg/day
murmur.
VSD: Pansystolic murmur ,Holosystolic Correct Answer: 3. 110–120 kcal/kg/day
murmur at lower left sternal border.
TOF: Systolic ejection murmur over Rationale:
pulmonary area, cyanosis common. Full-term newborns need about 110–120
kcal per kg per day to support growth and
Q. A 9-year-old child presents with maintain basal metabolic functions.
bedwetting at night for the past 6 months. Preterm infants may require slightly higher
There are no signs of urinary tract infection calories depending on clinical condition.
or neurological disorders. Which of the 50–60 kcal/kg/day is insufficient and may
following is the drug of choice for primary lead to growth failure.
nocturnal enuresis? 150–180 kcal/kg/day is excessive for a
healthy term newborn.
1.Imipramine
Q. A mother brings her 3-day-old newborn MINOR SUBJECT
to the clinic, worried because the baby has Q. A patient with hypertension has been
lost about 10% of birth weight. The baby is prescribed propranolol (a beta-blocker). As
otherwise feeding well and has normal a nursing officer, which of the following is
urine and stool output. What is the most the most important nursing responsibility
appropriate nursing advice? before administering the drug?

1. Advise immediate hospital admission for 1. Monitor serum potassium level


IV fluids 2. Assess blood pressure and apical pulse
2. Reassure the mother and encourage rate
frequent breastfeeding 3. Check blood glucose level
3. Stop breastfeeding and start formula 4. Evaluate liver function test
feeding
4. Suggest giving glucose water in between Correct Answer: 2. Assess blood pressure
feeds and apical pulse rate

Correct Answer: 2. Reassure the mother Rationale:


and encourage frequent breastfeeding Beta blockers reduce heart rate, blood
pressure, and myocardial contractility.
Rationale: The nursing officer must assess BP and
Normal weight loss in the first few days: apical pulse before administration.
Newborns typically lose 5–10% of birth If HR < 60 bpm or systolic BP < 100 mmHg,
weight in the first 3–5 days and regains on the medication should be withheld and the
10th day . physician notified.
If the baby is feeding well, alert, and has Other options (potassium, glucose, liver
adequate urine/stool output, this weight function) may be important in some
loss is physiologic. contexts but are not the priority before
Reassurance and frequent breastfeeding giving beta blockers.
(8–12 times/day) are the best interventions.
Hospitalization, formula, or glucose water Q. A 60-year-old patient receiving IV heparin
are not required unless there are signs of therapy develops bleeding from the IV site
dehydration or poor feeding. and gum bleeding. The doctor orders an
antidote to reverse heparin’s effect. Which
of the following drugs should the nursing
officer prepare to administer?

1. Vitamin K
2. Protamine sulfate
3. Fresh frozen plasma
4. Desmopressin

Correct Answer: 2. Protamine sulfate


Rationale: intravenously.” How much fluid in milliliters
Protamine sulfate is the specific antidote (mL) should the nurse prepare?
for heparin overdose or toxicity.
It neutralizes heparin’s anticoagulant effect 1. 250 mL
by forming a stable inactive complex. 2. 500 mL
Vitamin K is the antidote for warfarin, not 3. 1000 mL
heparin. 4. 750 mL
FFP may be used for multiple coagulation
factor deficiencies but is not the direct Correct Answer: 2. 500 mL
antidote.
Desmopressin is used for hemophilia A and Rationale:
von Willebrand disease. 1 pint (US) ≈ 473 mL, often rounded to 500
mL for clinical purposes.
Q. A 45-year-old man presents with pain 1 pint (UK/Imperial) ≈ 568 mL.
and swelling around the joint that bears the Accurate knowledge of volume conversions
maximum weight of the body and is most is essential for safe IV fluid administration.
commonly involved in degenerative
arthritis. This joint is also the largest Q. Which of the following statements
synovial joint in the human body. Which correctly describes the action of PTH and
joint is it? Calcitonin?

1. Shoulder joint 1. PTH increases serum calcium, while


2. Hip joint Calcitonin decreases serum calcium
3. Knee joint 2. Both PTH and Calcitonin increase serum
4. Elbow joint calcium levels
3. PTH decreases serum calcium, while
Correct Answer: 3. Knee joint Calcitonin increases serum calcium
4. Both PTH and Calcitonin decrease serum
Rationale: calcium levels
The knee joint is the largest synovial joint in
the human body. Correct Answer: 1. PTH increases serum
It is a modified hinge joint, formed between calcium, while Calcitonin decreases serum
the femur, tibia, and patella. calcium
It bears almost the entire body weight
during standing and movement. Rationale:
While the shoulder is the most mobile, the PTH (Parathyroid Hormone):
hip is the strongest, but the knee is the Increases serum calcium by stimulating
largest synovial joint, making it prone to bone resorption, enhancing calcium
injury and degenerative conditions like reabsorption in kidneys, and increasing
osteoarthritis. intestinal calcium absorption via Vitamin D
activation.
Q. A patient’s fluid order states “1 pint of Calcitonin (from thyroid C-cells):
normal saline to be administered
Lowers serum calcium by inhibiting 3. It ensures individualized learning for
osteoclastic bone resorption and promoting each student
calcium deposition in bone. 4. It replaces the need for discussion or
interactive sessions
Q. In nursing practice, understanding a
patient’s sociocultural background is Correct Answer: 2. It allows the instructor
important because: to efficiently deliver information to a large
group
1. It helps the nurse prescribe medications
more effectively Rationale:
2. It allows the nurse to provide culturally The lecture method is a teacher-centered
sensitive care and improve patient approach suitable for delivering large
compliance amounts of information quickly.
3. It ensures the patient will not develop any Advantages: time-efficient, organized
disease presentation of content, suitable for large
4. It replaces the need for clinical groups.
assessment Limitations: less interactive, limited
individualized attention, minimal hands-on
Correct Answer: 2. It allows the nurse to skill development.
provide culturally sensitive care and For skills or critical thinking, discussion,
improve patient compliance demonstration, and simulation methods are
more effective.
Rationale:
Sociology in nursing helps understand: Q. Which of the following stains is used to
Social norms, cultural beliefs, and values identify acid-fast bacteria, such as
Family dynamics and social support Mycobacterium tuberculosis, in clinical
systems specimens?
Factors influencing health behaviors and
adherence to treatment 1. Gram stain
This knowledge improves communication, 2. Ziehl-Neelsen stain
patient satisfaction, and health outcomes. 3. India ink stain
It does not replace clinical assessment or 4. Silver stain
directly prevent disease.
Correct Answer: 2. Ziehl-Neelsen stain
Q. Which of the following is the primary
advantage of using the lecture method in Rationale:
nursing education? Acid-fast bacteria (e.g., Mycobacterium
tuberculosis) have a waxy cell wall that
1. It promotes active hands-on skill practice resists decolorization by acid-alcohol.
for all students Ziehl-Neelsen stain is the standard method
2. It allows the instructor to efficiently to detect these bacteria, showing them as
deliver information to a large group red rods on a blue background.
Gram stain (A) is used for most bacteria but 3. Nursing is solely the documentation of
not acid-fast species. patient care and maintenance of hospital
India ink (C) detects capsules (e.g., records
Cryptococcus). 4. Nursing is limited to emergency care and
Silver stain (D) is used for certain fungi and surgical assistance
bacteria like Legionella.
Correct Answer: 2. Nursing is the
Q. Which of the following enzymes is autonomous and collaborative care of
responsible for breaking down individuals of all ages, families, groups, and
carbohydrates into simple sugars during communities, sick or well, to promote
digestion? health, prevent illness, and provide holistic
care
1. Pepsin
2. Amylase Rationale:
3. Lipase The ICN 2021 definition emphasizes:
4. Trypsin Autonomy and collaboration in care
Care across all ages and health states
Correct Answer: 2. Amylase Promotion of health, prevention of illness,
and holistic care
Rationale: Tasks like administering medication or
Amylase (salivary and pancreatic) catalyzes documentation are part of nursing but do
the breakdown of starch and glycogen into not fully define the profession.
maltose and dextrins.
Pepsin (A) digests proteins into peptides.
Lipase (C) digests fats into fatty acids and
glycerol.
Trypsin (D) is a pancreatic enzyme that
digests proteins in the small intestine.

Q. According to the International Council of


Nurses (ICN) 2021 update, which of the
following best defines nursing?

1. Nursing is primarily the task of


administering medications and treatments
prescribed by physicians
2. Nursing is the autonomous and
collaborative care of individuals of all ages,
families, groups, and communities, sick or
well, to promote health, prevent illness, and
provide holistic care
MENTAL HEALTH NURSING Verbigeration is the compulsive repetition
Q. A nursing officer is interviewing a of meaningless words or phrases, often
patient. When asked, “Do you have seen in schizophrenia.
children?” the patient replies: Echolalia is the repetition of another
"One is in college, he is studying person’s words.
engineering. He likes cricket, and last year Palilalia is the repetition of one’s own
he won a trophy. My other child is in school, words or phrases.
she likes painting… oh yes, so I have two Perseveration refers to the inappropriate
children." repetition of a previous response or action.
This is an example of:
Q. A patient with bipolar disorder is on
1. Tangentiality lithium therapy. The clinician monitors the
2. Circumstantiality serum lithium level to ensure safety and
3. Neologism efficacy. Which of the following is
4. Flight of ideas considered the usual therapeutic range for
lithium?
Correct Answer: 2. Circumstantiality
1. 0.2–0.6 mEq/L
Rationale: 2. 0.5–1.5 mEq/L
The patient gives irrelevant, unnecessary 3. 1.5–2.5 mEq/L
details but finally answers the question. 4. 2.0–3.0 mEq/L
In Tangentiality, the person would never
return to the main point. Correct Answer: 2. 0.5–1.5 mEq/L
Neologism = invention of new words.
Flight of ideas = rapid shifting ideas without Rationale:
full sentences, seen in mania. Lithium is therapeutic within 0.5–1.5 mEq/L,
depending on whether it’s for maintenance
Q. A 65-year-old patient with chronic or acute mania.
schizophrenia is observed repeating Levels above 1.5 mEq/L can lead to toxicity.
meaningless words and phrases Regular serum monitoring is critical during
persistently throughout the day, without treatment.
any apparent purpose. Which of the
following terms best describes this Q. A 35-year-old patient on chronic lithium
behavior? therapy presents with nausea, vomiting,
tremors, and confusion. Serum lithium is
1. Echolalia 2.0 mEq/L. Which of the following is the
2. Verbigeration most appropriate initial management of
3. Palilalia lithium toxicity?
4. Perseveration
1. Continue lithium and observe
Correct Answer: 2. Verbigeration 2. Stop lithium, ensure adequate hydration,
and remove unabsorbed lithium from the GI
Rationale: tract
3. Give IV potassium replacement only TFT: Lithium may induce hypothyroidism;
4. Administer high-dose insulin baseline thyroid function is recommended.
Pregnancy test: Lithium is teratogenic
Correct Answer: 2. Stop lithium, ensure (especially in the first trimester), so
adequate hydration, and remove pregnancy status must be known before
unabsorbed lithium from the GI tract starting therapy.

Rationale: Q. A 22-year-old college student reports


Step 1: Stop lithium immediately. intense fear of speaking in front of a class,
Step 2: Correct hydration and electrolyte avoids social gatherings, and experiences
imbalances. palpitations, sweating, and trembling during
Step 3: Remove unabsorbed lithium using social interactions. These symptoms have
polyethylene , glycol sodium polystyrene persisted for over 6 months and
sulphonate . significantly interfere with daily functioning.
Severe cases with very high lithium levels Which of the following is the most likely
or renal failure may require hemodialysis. diagnosis?
Supportive care and monitoring of renal
function, electrolytes, and cardiac status 1. Panic disorder
are essential. 2. Social anxiety disorder
3. Generalized anxiety disorder
Q. A 28-year-old patient is planned to start 4. Specific phobia
lithium therapy for bipolar disorder. Which
of the following baseline investigations is Correct Answer: 2. Social anxiety disorder
essential before initiating lithium?
Rationale:
1. Kidney function tests (KFT), ECG, thyroid Key features of Social Anxiety Disorder:
function tests (TFT), pregnancy test Persistent fear of social or performance
2. Liver function tests (LFT) and complete situations where the person may be
blood count (CBC) only scrutinized.
3. Serum electrolytes only Anxiety symptoms: palpitations, sweating,
4. Fasting blood sugar and lipid profile trembling, blushing.
Avoidance of social situations.
Correct Answer: 1. Kidney function tests Duration: ≥6 months.
(KFT), ECG, thyroid function tests (TFT), Significant impairment in daily life.
pregnancy test
Differentiation:
Rationale: Panic disorder: Recurrent unexpected panic
KFT: Lithium is renally excreted; baseline attacks, not limited to social situations.
kidney function is necessary. GAD: Excessive anxiety about multiple
ECG: Lithium can cause cardiac events, not just social performance.
arrhythmias, especially in elderly or Specific phobia: Fear is limited to a specific
patients with cardiac disease. object or situation, not generalized social
situations.
flashbacks, anxiety, insomnia, and
Q. A 16-year-old patient has an intense fear emotional numbness. His symptoms cause
of needles and avoids injections despite significant distress but have lasted for 3
needing vaccination. As a nursing officer, weeks. What is the most likely diagnosis?
which of the following is the most
appropriate nursing intervention in the 1. Post-Traumatic Stress Disorder (PTSD)
management of this specific phobia? 2. Acute Stress Disorder (ASD)
3. Adjustment Disorder
1. Encourage avoidance of injections until 4. Generalized Anxiety Disorder
the patient feels ready Correct Answer: 2. Acute Stress Disorder
2. Educate the patient about relaxation (ASD)
techniques and gradually expose them to
needles Rationale:
3. Prescribe benzodiazepines without Acute Stress Disorder:
counseling Occurs within 3 days to 4 weeks of
4. Ignore the fear and proceed with forced exposure to a traumatic event.
injection Symptoms: intrusive thoughts, flashbacks,
dissociation, anxiety, hyperarousal, sleep
Correct Answer: 2. Educate the patient disturbance.
about relaxation techniques and gradually Causes significant distress or impairment.
expose them to needles PTSD: Symptoms persist beyond 4 weeks.
Adjustment Disorder: Emotional or
Rationale: behavioral symptoms in response to a
Specific phobia management: stressor but do not meet criteria for
Non-pharmacological interventions are ASD/PTSD.
first-line, especially systematic GAD: Chronic anxiety about multiple life
desensitization (gradual exposure) and domains, not linked to a traumatic event.
relaxation techniques.
Nursing officer role: Q. A 30-year-old patient develops
Provide education about the phobia. flashbacks, nightmares, and hypervigilance
Support coping strategies (deep breathing, following a traumatic event 2 months ago.
guided imagery). What is the best initial management of
Assist in graded exposure in a safe, Post-Traumatic Stress Disorder (PTSD)?
supportive environment.
1. Long-term benzodiazepines
Pharmacotherapy is rarely first-line unless
2. Supportive psychotherapy and
severe anxiety occurs.
trauma-focused cognitive behavioral
Avoid forcing the patient, as it can worsen
therapy (CBT)
fear and mistrust.
3. Immediate hospitalization and restraints
4. Ignoring the symptoms as they will
resolve spontaneously
Q. A 25-year-old man witnesses a serious
Correct Answer: 2. Supportive
road traffic accident. Over the next 2 days,
psychotherapy and trauma-focused
he experiences intrusive memories,
cognitive behavioral therapy (CBT)
electroconvulsive therapy (ECT). Which of
Rationale: the following statements is correct
First-line treatment of PTSD: regarding ECT?
Psychotherapy: Trauma-focused CBT or
exposure therapy is most effective. 1. ECT is only used when medications fail
Supportive counseling: Helps the patient 2. ECT can cause permanent memory loss
cope with stress and regain functioning. in all patients
Pharmacotherapy (SSRIs) may be added if 3. ECT is safe in pregnancy and can be
symptoms are severe, but psychotherapy life-saving
remains primary. 4. ECT is contraindicated in all
Benzodiazepines are generally avoided due cardiovascular conditions
to dependence risk and limited efficacy. Correct Answer: 3. ECT is safe in
Hospitalization or ignoring symptoms is not pregnancy and can be life-saving
appropriate in stable outpatients.
Rationale:
Q. Which of the following is the most Indications of ECT:
common psychiatric comorbidity Severe depression with psychotic features
associated with Obsessive-Compulsive Suicidal ideation or refusal to eat
Disorder (OCD)? Treatment-resistant cases
Mania with severe agitation
1.Schizophrenia Catatonia
2. Major Depressive Disorder Safety:
3. Bipolar Disorder ECT is considered safe in pregnancy,
4. Social Anxiety Disorder especially in the 2nd trimester.
Side effects: Short-term memory loss and
Correct Answer: 2. Major Depressive confusion are common; permanent memory
Disorder loss is rare.
Precautions: Cardiovascular diseases
Rationale: require monitoring, but not an absolute
OCD comorbidities: contraindication.
The most common is Major Depressive
Q. Identify the condition shown in the
Disorder (MDD), occurring in about 50–60%
image:
of cases.
Other comorbidities may include anxiety
disorders (e.g., social anxiety), tic
disorders, and personality disorders, but
depression is the most frequent.
Clinical importance: Comorbidity worsens
prognosis and may influence treatment
choices.
Q. A 35-year-old patient with severe,
1. Phytobezoar
treatment-resistant depression and
2. Trichobezoar
psychotic features is considered for
3. Pancreatic pseudocyst
4. Gastrointestinal stromal tumor (GIST) 2. Competing response training – perform a
voluntary behavior incompatible with the
Correct Answer: 2. Trichobezoar habit.
3. Social support – family or peers help
Rationale: reinforce therapy.
Trichobezoar: 4. Relaxation techniques – manage stress
Hair accumulation in the stomach, that triggers the habit.
sometimes extending into the intestine
Other options:
(Rapunzel syndrome).
Psychoeducation alone is supportive but
Common in adolescent girls with
insufficient.
trichotillomania and trichophagia.
ECT or benzodiazepines are not indicated
Imaging features: Well-defined intragastric
for habit disorders.
mass, mottled or heterogeneous
appearance on X-ray/CT. Q. A 19-year-old male repeatedly steals
Phytobezoar: Plant material, often after items he does not need and feels tension
gastric surgery, but patient history differs. before stealing and relief afterward, without
GIST/pseudocyst: Usually solid or any intent for personal gain. Another
fluid-filled mass without hair history. patient, a 21-year-old male, has repeated
episodes of setting fires, feels tension
Q. A 12-year-old girl has been pulling her before and pleasure after the act, and has
hair for the past year, causing noticeable no external motive. Which of the following
hair loss. She is distressed and wants help statements is correct regarding these
to stop this behavior. Which of the following conditions?
is the most appropriate first-line behavioral
intervention? 1. Both are impulse control disorders
characterized by failure to resist an urge
1. Psychoeducation only despite negative consequences
2. Habit Reversal Therapy (HRT) 2. Kleptomania is motivated by financial
3. Electroconvulsive therapy (ECT) gain
4. Long-term benzodiazepines 3. Pyromania is a form of depression
4. Both conditions are primarily treated with
Correct Answer: 2. Habit Reversal Therapy electroconvulsive therapy (ECT)
(HRT)
Correct Answer: 1. Both are impulse control
disorders characterized by failure to resist
Rationale:
an urge despite negative consequences
Habit Reversal Therapy:
First-line behavioral therapy for
Rationale:
trichotillomania, nail-biting, and other
Kleptomania: Recurrent stealing without
repetitive habits.
need or personal gain, preceded by tension
and followed by relief.
Key components:
Pyromania: Recurrent deliberate
1. Awareness training – recognize the urge
fire-setting, tension before, pleasure after,
and triggers.
no external motive.
Both are classified as Impulse Control The Lorazepam challenge test is a rapid
Disorders (ICDs) in DSM-5. diagnostic tool for catatonia.
Treatment: Behavioral therapy (habit Negative symptoms of schizophrenia and
reversal, CBT), SSRIs for comorbid depression may show psychomotor
anxiety/depression; ECT is not indicated.
slowing but won’t respond acutely to
Key feature: impulse control failure, not
lorazepam.
external reward or depression. Dissociative stupor has a psychological
basis and also won’t show such a
Q. A patient with conversion disorder pharmacological response.
shows little concern about their sudden
paralysis. This feature is called:

1. Anosognosia
2. La Belle Indifférence
3. Malingering
4. Catatonia

Correct Answer: 2. La Belle Indifférence

Rationale:
La Belle Indifférence: Calm or unconcerned
attitude toward serious symptoms,
commonly seen in conversion disorder.
Key point: The patient’s lack of distress is
out of proportion to the severity of
symptoms.

Q. A 35-year-old patient presents with


mutism, immobility, and posturing for the
past 3 days. The psychiatrist administers 2
mg IV lorazepam, and within 10 minutes the
patient starts speaking and moving. This
dramatic response is diagnostic of:

1. Catatonia
2. Schizophrenia – negative symptoms
3. Major depressive disorder with
psychomotor retardation
4. Dissociative stupor

Correct Answer: 1. Catatonia

Rationale:

You might also like